Тесты по клинической фармакологии для студентов. Предэкзаменационный тест по фармакологии

Тест по Клинической фармакологии

система подготовки к тестам Gee Test oldkyx.com

Список вопросов по Клинической фармакологии

1. Период полувыведения лекарства - это:
1) [-] время достижения максимальной концентрации лекарства в плазме;

2) [-] время, в течение которого лекарство достигает системного кровотока;

3) [-] время, в течение которого лекарство распределяется в организме;

4) [+] время, за которое концентрация лекарства в плазме снижает¬ся на 50%;

5) [-] время, за которое половина введенной дозы достигает орга¬на-мишени.

2. Широта терапевтического действия - это:
1) [-] терапевтическая доза лекарства;

2) [-] отношение концентрации лекарства в органе или ткани к концентрации его в плазме крови;

3) [+] диапазон между минимальной терапевтической и минималь¬ной токсической концентрациями лекарства в плазме;

4) [-] процент не связанного с белком лекарства;

5) [-] диапазон между минимальной и максимальной терапевтиче¬скими концентрациями лекарства.

3. К рецепторным средствам конкурентного действия относятся:
1) [-] НПВС (нестероидные противовоспалительные средства);

2) [+] β-адреноблокаторы;

3) [-] петлевые диуретики;

4) [-] нитраты;

5) [-] фторхинолоны.

4. Функцию печени и почек следует учитывать при назначении следующих лекарственных средств:
1) [-] липофильных, образующих неактивные метаболиты;

2) [+] липофильных, образующих активные метаболиты;

3) [-] гидрофильных;

4) [-] гепатотоксичных;

5) [-] нефротоксичных.

5. Селективность действия лекарственного вещества зависит от:
1) [-] периода полувыведения;

2) [-] способа приема;

3) [-] связи с белком;

4) [-] объема распределения;

5) [+] дозы.

6. Для кинетики насыщения характерно:
1) [+] увеличение периода полувыведения введенной дозы при не¬измененном клиренсе;

2) [-] скорость элиминации пропорциональна концентрации препа¬рата в плазме и дозе;

3) [-] период полувыведения не пропорционален введенной дозе.

7. Фактор, определяющий необходимость перерасчета режима введения препарата при ХПН:
1) [-] высокая липофильность препарата;

2) [-] низкая связь с белками плазмы;

3) [-] наличие систем активного канальцевого пути экскреции;

4) [+] высокая степень экскреции в неизмененном виде.

8. Какие препараты с большей легкостью проходят через ГЭБ?
1) [-] с высокой растворимостью в воде;

2) [+] с высокой растворимостью в в жирах;

3) [-] проявляющие свойства слабых кислот;

4) [-] проявляющие свойства слабых оснований;

5) [-] со слабой связью с белками плазмы.

9. В каком случае происходит более полное всасывание?
1) [-] всасывание из желудка препарата, проявляющего свойства слабого основания;

2) [-] всасывание из тонкого кишечника препарата, проявляющего свойства слабой кислоты;

3) [+] всасывание из тонкого кишечника препарата, проявляющего свойства слабого основания.

10. В понятие «пресистемный метаболизм» входит:
1) [+] биотрансформация препаратов в печени при первом прохож¬дении и в кишечнике;

2) [-] биотрансформация препаратов в кишечнике;

3) [-] биотрансформация препаратов в печени при первом прохож¬дении и в почках;

4) [-] биотрансформация препаратов в печени, в почках и в кишечнике.

11. Строго дозозависимой является следующая группа побочных эффектов:
1) [-] фармацевтические;

2) [-] фармакогенетические;

3) [-] аллергические;

4) [-] мутагенные;

5) [+] синдром отмены.

12. Определите группу препаратов с узким терапевтическим индексом:
1) [-] β-блокаторы;

2) [-] пенициллины;

3) [+] сердечные гликозиды;

4) [-] ингибиторы АПФ;

5) [-] мощные диуретики.

13. Проведение лекарственного мониторинга желательно при лечении следующей группой препаратов:
1) [+] противосудорожными;

2) [-] β2-симптомомиметиками;

3) [-] пенициллинами;

4) [-] глюкокортикоидами;

5) [-] М-холинолитиками.

14. К отсроченным относится следующая группа побочных эффектов:
1) [-] токсические;

2) [-] развитие лекарственной зависимости;

3) [-] фармакогенетические;

4) [+] канцерогенные;

5) [-] синдром отмены.

15. Развитие асистолии возможно при комбинации пропранолола с:
1) [-] фенобарбиталом;

2) [-] фуросемидом;

3) [+] верапамилом;

4) [-] фенитоином;

5) [-] ранитидином.

16. Риск токсических эффектов увеличивается при комбинации гентамицина с:
1) [+] фуросемидом;

2) [-] пенициллином;

3) [-] метилксантинами;

4) [-] макролидами;

5) [-] глюкокортикоидами.

17. Риск развития нежелательной беременности увеличивается при сочетании оральных контрацептивов с:
1) [-] гипотензивными;

2) [-] витамином С;

3) [-] алкоголем;

4) [+] тетрациклином;

5) [-] глюкокортикоидами.

18. При патологии почек возникают следующие изменения фармакокинетики лекарств, кроме:
1) [-] нарушения почечной экскреции;

2) [-] увеличения концентрации лекарств в плазме крови;

3) [-] уменьшения связывания с белками плазмы;

4) [-] увеличения Т1/2;

5) [+] уменьшения биодоступности.

19. Цирроз печени вызывают следующие изменения фармакокинетики лекарств, кроме:
1) [-] снижения пресистемного метаболизма;

3) [-] увеличения Т1/2;

4) [-] увеличения биодоступности;

5) [+] уменьшения объема распределения.

20. При сердечной недостаточности наблюдаются следующие изменения фармакокинетики дигоксина, кроме:
1) [-] снижения абсорбции в ЖКТ на 30%;

2) [-] уменьшения связывания с белками плазмы;

3) [+] усиления метаболизма в печени;

4) [-] снижения почечной экскреции;

5) [-] увеличения Т1/2.

21. Алкоголь при однократном приеме больших доз приводит к:
1) [-] увеличению абсорбции лекарств;

3) [+] замедлению метаболизма в печени;

4) [-] снижению почечной экскреции;

5) [-] увеличению Т1/2.

22. Никотин приводит к:
1) [-] уменьшению абсорбции лекарств;

2) [-] увеличению объема распределения лекарств;

3) [-] уменьшению связи с белком плазмы;

4) [+] усилению метаболизма в печени;

5) [-] усилению почечной экскреции лекарств.

23. Обычной формой высвобождения лекарственного вещества характеризуется:
1) [-] нитронг;

2) [-] сустак-мите;

3) [+] нитросорбид;

4) [-] нифедипин-GITS;

5) [-] верапамил SR.

24. Для купирования приступа стенокардии применяют сублингвально таблетированную лекарственную форму:
1) [-] нитронг;

2) [-] сустак;

3) [+] нитросорбид;

4) [-] атенонолол;

5) [-] верапамил SR.

25. Для предотвращения развития толерантности при регулярном применении нитратов безнитратный интервал должен составлять:
1) [-] 2-4 часа;

2) [-] 4-6 часов;

3) [-] 6-8 часов;

4) [+] 8-12 часов.

26. Для усиления антиангинального эффекта наиболее безопасным является сочетание:
1) [-] верапамил + пропранолол;

2) [-] верапамил + атенолол;

3) [-] верапамил + метопролол;

4) [+] верапамил + изосорбида динитрат;

5) [-] верапамил + дилтиазем.

27. Методами оценки антиангинальной эффективности препарата являются все перечисленные, кроме:
1) [-] холтеровского мониторирования ЭКГ;

2) [+] мониторирования суточного АД;

3) [-] стресс-Эхо;

4) [-] тредмил-теста;

5) [-] ВЭМ-пробы.

28. У больного стенокардией в сочетании с артериальной гипертонией имеют преимущество препараты:
1) [-] нитраты;

29. При стенокардии напряжения препаратами выбора являются препараты следующего класса:

2) [+] блокаторы β-адренорецепторов;

3) [-] блокаторы α-адренорецепторов;

4) [-] агонисты имидазолиновых рецепторов;

5) [-] блокаторы рецепторов ангиотензина II.

30. При вазоспастической стенокардии препаратами выбора являются препараты следующего класса:
1) [-] блокаторы гистаминовых рецепторов;

2) [-] блокаторы β-адренорецепторов;

3) [-] блокаторы α-адренорецепторов;

4) [+] блокаторы кальциевых каналов;

5) [-] блокаторы рецепторов ангиотензина II.

31. Адекватным методом контроля за эффективностью и безопасностью гипотензивной терапии является:
1) [-] суточное мониторирование ЭКГ;

2) [+] суточное мониторирование АД;

3) [-] разовые измерения АД;

4) [-] измерение показателей ФВД;

5) [-] динамика интервала QT на ЭКГ.

32. Выберите нежелательный эффект, не характерный для верапамила:
1) [-] брадикардия;

2) [-] запоры;

3) [-] развитие AV-блокады;

4) [-] отеки голеней и стоп;

5) [+] бронхоспазм.

33. Для лечения артериальной гипертензии препаратом первого выбора у больного хронической сердечной недостаточностью является:
1) [+] эналаприл;

2) [-] верапамил;

3) [-] клофелин;

4) [-] празозин;

5) [-] нифедипин.

34. Укажите гипотензивный препарат, увеличивающий активность симпатоадреналовой системы:
1) [+] нифедипин;

2) [-] клофелин;

3) [-] каптоприл;

4) [-] метопролол;

5) [-] ирбесартан.

35. При артериальной гипертонии в сочетании с синусовой тахикардией предпочтение следует отдать:
1) [-] блокаторам кальциевых каналов производных дигидропиридина;

2) [-] петлевым диуретикам;

3) [+] β-адреноблокаторам;

4) [-] α-адреноблокаторам;

5) [-] тиазидным диуретикам.

36. α1-адреноблокаторы являются препаратами выбора для лечения артериальной гипертонии:
1) [-] у пациентов с заболеваниями печени;

2) [-] у больных с нарушениями ритма;

3) [+] у пожилых мужчин с аденомой предстательной железы и за¬трудненным мочеиспусканием;

4) [-] у пациентов со стенокардией;

5) [-] у больных с инфарктом миокарда в анамнезе.

37. Для лечения артериальной гипертонии у больных с бронхиальной астмой не могут быть использованы:
1) [-] блокаторы кальциевых каналов;

2) [-] антагонисты рецепторов к ангиотензину II;

3) [-] α1-адреноблокаторы;

4) [+]β-адреноблокаторы;

5) [-] диуретики.

38. Пациентам с артериальной гипертонией и инфарктом миокарда в анамнезе в первую очередь следует назначить:
1) [+] β-адреноблокаторы;

2) [-] диуретики;

3) [-] блокаторы кальциевых каналов;

4) [-] агонисты имидазолиновых рецепторов;

5) [-] α1-адреноблокаторы.

39. Препаратами первого выбора у пациентов с артериальной гипертонией и выраженным стенозом периферических артерий являются:
1) [-] неселективные β-адреноблокаторы;

2) [+] антагонисты кальция;

3) [-] диуретики;

4) [-] блокаторы рецепторов к ангиотензину II;

5) [-] агонисты α2-адренорецепторов.

40. Перечислите группы препаратов, улучшающих прогноз пациентов с ХСН:
1) [-] ингибиторы АПФ;

2) [-] β-блокаторы;

3) [-] блокаторы рецепторов к ангиотензину II;

4) [-] спиронолактон;

41. Перечислите препараты, обладающие прямым положительным инотропным эффектом:
1) [-] дигоксин;

2) [-] допамин;

3) [-] амринон;

4) [-] левосимендан;

5) [+] все перечисленные препараты.

42. Перечислите β-блокаторы, обладающие доказанной эффективностью в лечении пациентов с ХСН:
1) [-] атенолол;

2) [-] пропранолол;

3) [+] карведилол;

4) [-] соталол;

43. Укажите показания к назначению спиронолактона:
1) [-] уровень калия в крови > 5, 5 ммоль/л;

2) [-] отеки голеней и стоп;

3) [+] сердечная недостаточность IV ФК по классификации NYHA;

4) [-] клиренс креатинина менее 30 мл/мин;

44. Титрование дозы ингибиторов АПФ и β-блокаторов у пациентов с ХСН подразумевает:
1) [-] начало терапии с минимальной дозы препарата;

2) [-] увеличение дозы препарата каждые 2 недели;

3) [-] достижение целевой дозы препарата;

4) [-] снижение количества госпитализаций и увеличение продол¬жительности жизни больного;

5) [+] все перечисленные признаки.

45. Показания для назначения амлодипина при ХСН:
1) [-] застойная сердечная недостаточность;

2) [+] неконтролируемые цифры артериального давления;

3) [-] инфаркт миокарда в анамнезе;

4) [-] нарушения ритма;

5) [-] все перечисленные признаки.

46. Принципы диуретической терапии при ХСН:
1) [-] назначение диуретиков при сердечной недостаточности II-IV ФК по классификации NYHA;

2) [-] снижение веса на 0,5-1,0 кг в сутки;

3) [-] контроль артериального давления;

4) [-] контроль за уровнем калия в крови;

5) [+] все перечисленные.

47. Укажите предпочтительный путь введения лекарственных препаратов при застойной сердечной недостаточности:
1) [-] сублингвальный;

2) [-] ректальный;

3) [+] парентеральный;

4) [-] пероральный;

5) [-] все перечисленные пути введения.

48. Эффекты ингибитора АПФ у пациентов с ХСН:
1) [-] влияние ингибитора АПФ на смертность зависит от продол¬жительности лечения;

2) [-] снижение риска летального исхода более выражено у паци¬ентов с более высоким ФК;

3) [-] наличие дозозависимого эффекта ингибитора АПФ у боль¬ных сердечной недостаточностью;

4) [+] все перечисленные.

49. Перечислите препараты, увеличивающие продолжительность потенциала действия:
1) [-] хинидин;

2) [-] прокаинамид;

3) [-] амиодарон;

4) [-] дигоксин;

5) [+] все перечисленные препараты.

50. Препараты, удлиняющие интервал QT:
1) [-] клиндамицин;

2) [-] амиодарон;

3) [-] котримоксазол;

4) [-] хинидин;

5) [+] все перечисленные препараты.

51. Препараты, удлиняющие интервал PQ:
1) [-] лидокаин;

2) [-] хинидин;

3) [-] дизопирамид;

4) [+] дигоксин;

5) [-] все перечисленные препараты к назначению антиаритмических препаратов.

52. Показаниями к назначению антиаритмических препаратов являются:
1) [-] частые нарушения ритма;

2) [-] желудочковые экстрасистолы - 6 в минуту;

3) [-] нарушение ритма высоких градаций;

4) [+] нарушение гемодинамики;

5) [-] все перечисленные.

53. Перечислите препараты, имеющие холинолитические побочные эффекты:
1) [-] амиодарон;

2) [-] веропамил;

3) [-] лидокаин;

4) [+] хинидин;

5) [-] все перечисленные препараты.

54. Перечислите препараты, повышающие порог фибрилляции:
1) [-] кордарон;

2) [-] бретилий тозилат;

3) [-] соталол;

4) [-] пропранолол;

5) [+] все перечисленные препараты.

55. Перечислите препараты, назначаемые для профилактики нарушений ритма при WPW-синдроме:
1) [-] дигоксин;

2) [-] дилтиазем;

3) [+] амиодарон;

4) [-] прокаинамид;

5) [-] все перечисленные препараты.

56. Показания к восстановлению ритма при постоянной форме мерцательной аритмии:
1) [-] частые эпизоды тахисистолии;

2) [-] слабость синусового узла;

3) [+] наличие тромбоэмболии в анамнезе;

4) [-] неэффективность проводимой медикаментозной терапии;

5) [-] все перечисленные.

57. Препарат выбора для лечения пароксизмов желудочковой тахикардии:
1) [-] лидокаин;

2) [-] прокаинамид;

3) [-] бретилий;

4) [-] пропафенон;

5) [+] все перечисленные препараты.

58. Побочные эффекты амиодарона:
1) [-] фотосенсибилизация;

2) [-] нарушение функции щитовидной железы;

3) [-] сухой кашель;

4) [-] преходящее повышение активности печеночных аминотрансфераз;

5) [+] все перечисленные эффекты.

59. Показания к применению аденозина:
1) [-] пароксизм мерцательной аритмии;

2) [+] реципрокные наджелудочковые тахикардии;

3) [-] желудочковая тахикардия;

4) [-] экстрасистолия;

5) [-] все перечисленные.

60. Метод оценки эффективности антиаритмической терапии в амбулаторной практике:
1) [+] мониторинг ЭКГ по Холтеру;

3) [-] пробы с физической нагрузкой;

4) [-] ЭФИ (электорофизиологическое исследование);

5) [-] все перечисленные методы.

61. К ингаляционным глюкокортикоидам относится:
1) [-] гидрокортизон;

2) [+] беклометазон;

3) [-] преднизолон;

4) [-] полькорталон;

5) [-] дексаметазон.

62. К селективным β2-агонистам длительного действия относится:
1) [-] флутиказон;

2) [+] сальметерол;

3) [-] сальбутамол;

4) [-] фенотерол;

5) [-] тербуталин.

63. Для купирования приступа бронхиальной астмы применяется:
1) [-] тиотропиум бромид;

2) [-] теопэк;

3) [-] кромогликат натрия;

4) [+] сальбутамол;

5) [-] будесонид.

64. К холинолитикам длительного действия относят:
1) [-] ипратропиум бромид;

2) [-] кромгликат натрия;

3) [+] тиотропиум бромид;

4) [-] окситропиум бромид;

5) [-] триамцинолон ацетонид.

65. Побочным эффектом ингаляционных глюкокортикостероидов является:
1) [-] головная боль;

2) [-] ожирение;

3) [+] кандидоз полости рта;

4) [-] сахарный диабет;

5) [-] полиурия.

66. К муколитическим средствам относится:
1) [-] кодеин;

2) [-] кромогликат натрия;

3) [+] ацетилцистеин;

4) [-] сальметерол;

5) [-] теофиллин.

67. При одновременном применении повышает концентрацию теофиллина в крови:
1) [+] офлоксацин;

2) [-] пенициллин;

3) [-] цефтриаксон;

4) [-] гентамицин;

5) [-] бисептол.

68. При одновременном применении снижает концентрацию теофиллина в крови:
1) [-] пефлоксацин;

2) [-] циметидин;

3) [+] рифампицин;

4) [-] эритромицин;

5) [-] ампиокс.

69. К бронхолитикам не относятся:
1) [-] метилксантины;

2) [-] холинолитики;

3) [-] симпатомиметики;

4) [+] блокаторы лейкотриеновых рецепторов.

70. Препаратом базисной терапии при хроническом обструктивном бронхите является:
1) [+] тиотропиум бромид;

2) [-] недокромил натрия;

3) [-] фенотерол;

4) [-] монтелукаст;

5) [-] эуфиллин.

71. Тахикардия как побочный эффект развивается при приеме всех перечисленных препаратов, кроме:
1) [-] сальбутамола;

2) [-] изопротеринола;

3) [-] фенотерола;

4) [-] теофиллина;

5) [+] ипратропиума бромида.

72. К комбинированным средствам для лечения бронхиальной астмы не относят:
1) [-] дитек;

2) [-] серетид;

3) [-] симбикорт;

4) [+] беклазон;

5) [-] беродуал.

73. Наиболее выраженным противовоспалительным эффектом обладает ингаляционный глюкокортикостероид:
1) [-] бекламетазона дипропионат;

2) [-] будесонид;

3) [-] триамцинолона ацетонид;

4) [+] флутиказона пропионат;

5) [-] флунизолид.

74. Препаратом выбора при наличии инфекций, вызванных метициллинрезистентными штаммами золотистого стафилококка, является:
1) [-] азитромицин;

2) [-] метронидазол;

3) [-] гентамицин;

4) [+] линезолид;

5) [-] цефуроксим.

75. Выберите группу антибактериальных препаратов для лечения инфекций, вызванных внутриклеточными возбудителями:
1) [+] макролиды;

2) [-] пенициллины;

3) [-] аминогликозиды;

4) [-] цефалоспорины;

5) [-] сульфаниламиды.

76. Укажите группу антибактериальных препаратов, обладающих наибольшей антианаэробной активностью:
1) [-] гликопептиды;

2) [-] аминопенициллины;

3) [-] тетрациклины;

4) [-] аминогликозиды;

5) [+] нитроимидазолы.

77. Нефротоксичными являются все перечисленные антибактериальные препараты, кроме:
1) [-] гентамицина;

2) [-] карбенициллина;

3) [+] азитромицина;

4) [-] цефазолина;

5) [-] ванкомицина.

78. Укажите антибактериальный препарат, малоактивный в отношении пневмококка:
1) [-] азитромицин;

2) [-] пенициллин;

3) [-] цефтриаксон;

4) [+] ципрофлоксацин;

5) [-] левомицетин.

79. Выберите рациональную комбинацию антибактериальных препаратов, обладающую синергизмом действия в отношении грамположительных микроорганизмов и безопасностью:
1) [-] пенициллины + тетрациклины;

2) [-] пенициллины + цефалоспорины;

3) [-] аминогликозиды + гликопептиды;

4) [+] пенициллины + аминогликозиды;

5) [-] пенициллины + сульфаниламиды.

80. Хорошо проникают через гематоэнцефалический барьер следующие антибактериальные препараты:
1) [-] линкозамиды;

2) [-] макролиды;

3) [-] тетрациклины;

4) [-] аминогликозиды;

5) [+] цефалоспорины III генерации.

81. Препаратом выбора при крупозной пневмонии является:
1) [-] ципрофлоксацин;

2) [-] доксициклин;

3) [-] гентамицин;

4) [-] цефотаксим;

5) [+] бензилпенициллин.

82. Препаратом выбора при тонзиллофарингите является:
1) [+] цефуроксим аксетил;

2) [-] доксициклин;

3) [-] цефтазидим;

4) [-] офлоксацин;

5) [-] фурагин.

83. Препаратами выбора при инфекциях желчевыводящих путей являются:
1) [-] аминогликозиды;

2) [-] нитрофураны;

3) [+] цефалоспорины III поколения;

4) [-] макролиды;

5) [-] природные пенициллины.

84. При госпитальных инфекциях, возникших в реанимационных отделениях, препаратами выбора являются следующие комбинации антибактериальных препаратов:
1) [-] ампициллин + гентамицин;

2) [+] цефтазидим + амикацин;

3) [-] цефуроксим + эритромицин;

4) [-] клиндамицин + гентамицин;

5) [-] норфлоксацин + пенициллин.

85. Предпочтительной группой антибактериальных препаратов при лечении хронического простатита являются:
1) [-] сульфаниламиды;

2) [-] карбапенемы;

3) [-] хинолоны;

4) [-] линкозамины;

5) [+] фторхинолоны.

86. Выберите препарат, максимально подавляющий секрецию соляной кислоты:
1) [-] пирензепин;

2) [-] циметидин;

3) [-] мизопростол;

4) [-] антациды;

5) [+] омепразол.

87. Максимальное количество побочных эффектов среди Н2-блокаторов имеет:
1) [+] циметидин;

2) [-] роксатидин;

3) [-] низатидин;

4) [-] ранитидин;

5) [-] фамотидин.

88. Угнетает цитохром Р-450:
1) [-] омепразол;

2) [-] пирензепин;

3) [+] циметидин;

4) [-] фамотидин;

5) [-] лансопразол.

89. Синдром отдачи вызывают:
1) [-] синтетические простагландины;

2) [-] антациды;

4) [-] М-холинолитики;

5) [+] Н2-блокаторы.

90. При язвенной болезни двенадцатиперстной кишки рационально назначать антациды:
1) [-] до еды;

2) [-] во время еды;

3) [+] через 1, 5-2 часа после еды;

4) [-] через 5 часов после еды;

5) [-] вне зависимости от приема пищи.

91. Для профилактики язв, вызванных приемом НПВС, наиболее эффективны:
1) [-] антациды;

2) [-] Н2-блокаторы;

3) [-] блокаторы «протоновой помпы»;

4) [+] синтетические простагландины;

5) [-] М-холинолитики.

92. В эрадикационной терапии для уничтожения Н. pylori используется следующий антибиотик:
1) [-] карбенициллин;

2) [-] эритромицин;

3) [-] цефоперазон;

4) [+] кларитромицин;

5) [-] хлорамфеникол.

93. Большое практическое значение имеет развитие устойчивости Н. pylori к:
1) [-] ванкомицину;

2) [+] метронидазолу;

3) [-] тетрациклину;

4) [-] нитрофуранам;

5) [-] цефотаксиму.

94. Бактерицидным действием против Н. pylori обладает:
1) [-] сукральфат (вентер);

2) [+] субцитрат висмута (де-нол);

3) [-] альмагель;

4) [-] фамотидин;

5) [-] пирензепин.

95. В эрадикационной терапии для уничтожения Н. pylori используются:
1) [-] антациды;

2) [-] синтетические простагландины;

3) [+] блокаторы «протоновой помпы»;

4) [-] М-холинолитики;

5) [-] гликопептиды.

96. Кортикостероиды усиливают токсичность:
1) [-] теофиллина;

2) [+] тиазидных диуретиков;

3) [-] препаратов золота;

4) [-] противоязвенных средств.

97. Эффект глюкокортикоидов снижает:
1) [-] циметидин;

2) [-] аспирин;

3) [-] диклофенак;

4) [-] амиодарон;

5) [+] рифампицин.

98. Наиболее высокой минералокортикоидной активностью обладает:
1) [-] полькартолон;

2) [-] преднизолон;

3) [+] гидрокортизон;

4) [-] дексаметазон.

99. После приема НПВС быстрее развивается эффект:
1) [-] противовоспалительный;

2) [+] анальгетический;

3) [-] антикоагуляционный.

100. Наиболее выраженным анальгетическим свойством обладает:
1) [-] ацетилсалициловая кислота;

2) [-] ибупрофен;

3) [-] напроксен;

4) [+] парацетамол.

101. Механизмом развития ульцирогенного эффекта НПВС является:
1) [-] повышение кислотности желудочного сока;

2) [+] снижение синтеза простагландинов в слизистой оболочке же¬лудка;

3) [-] снижение репарации слизистой оболочки.

102. Наиболее выраженным противовоспалительным свойством обладает:
1) [+] фенилбутазон;

2) [-] метамизол;

3) [-] пироксикам;

4) [-] парацетамол;

5) [-] ибупрофен.

103. Ранним побочным эффектом глюкокортикоидов является:
1) [-] катаракта;

2) [-] миопатия;

3) [-] остеопороз;

4) [-] кушингоидный синдром;

5) [+] стероидный диабет.

104. Укажите эффект, не характерный для кортикостероидов:
1) [-] противовоспалительный;

2) [-] противоаллергический;

3) [-] противошоковый;

4) [-] иммунодепрессивный;

Предварительный просмотр:

Тема: «ЛРС, влияющие на периферическую нервную систему»

Тестовые задания

1.Адреналин вызывает:

Выберите один ответ.

a.) Снижение потребления кислорода

B.) Гипергликемию

C.) Угнетение гликогенолиза

D.) Угнетение липолиза

2.Адреналин противопоказан при:

Выберите один ответ.

a.) Тиреотоксикозе

B.) Анафилактическом шоке

C.) Сердечной блокаде

D.) Гипогликемической коме

3.Ганглиоблокатор:

Выберите один ответ.

a.) атропин;

B.) пипекуроний;

C.) пентамин;

D.) сукцинилхолин (дитилин).

4.Ганглиоблокаторы используются для лечения:

Выберите один ответ.

a.) запора.

B.) гипертензивного криза;

C.) глаукомы;

D.) задержки мочеотделения;

5.Действие агониста М-ХР блокируется:

Выберите один ответ.

a.) Цитизином

B.) Тубокурарином

C.) Прозерином

D.) Атропином

E.) Пилокарпином

6.Для прекращения действия конкурентных миорелаксантов применяется:

Выберите один ответ.

a.) атропин;

B.) дипироксим.

C.) неостигмин (прозерин);

7.Избирательный М-холиномиметик (агонист мускариновых холинорецепторов):

Выберите один ответ.

a.) Прозерин

B.) Пилокарпин

C.) Цитизин

D.) Физостигмин

E.) Карбахолин

8.К адсорбирующим средствам относится:

Выберите один ответ.

a.) Слизь крахмала.

B.) Отвар коры дуба;

C.)Танин;

D.) Уголь активированный;

9.К раздражающим средствам относится все кроме:

Выберите один ответ.

a.) Висмута нитрат основной;

B.) Ментол.

C.) Масло терпентинное очищенное (скипидар);

D.) Горчичная бумага;

10.М-холиноблокатор:

Выберите один ответ.

a.) пентамин;

B.) пипекуроний;

C.) сукцинилхолин (дитилин).

D.) атропин;

11.М-холиноблокаторы вызывают развитие мидриаза:

Выберите один ответ.

a.) повышая тонус радиальной мышцы радужной оболочки;

B.) снижая тонус круговой мышцы радужной оболочки;

C.) повышая тонус цилиарной мышцы.

12.М-холиноблокаторы используются для лечения:

Выберите один ответ.

a.) артериальной гипертензии;

B.) глаукомы;

C.) язвенной болезни желудка.

D.) миастении;

13.М-холиноблокаторы противопоказаны при:

Выберите один ответ.

a.) бронхиальной астме;

B.) глаукоме;

C.) атрио-вентрикулярной блокаде;

D.) язвенной болезни желудка.

14.М-холиномиметики, в отличие от ингибиторов ХЭ, не оказывают влияния на холинергическую синаптическую передачу:

Выберите один ответ.

a.) в нервно-мышечном синапсе

B.) с постганглионарных аксонов вегетативных нервов на эффектор (гладкая мышца, экзокринные железы)

C.) в ЦНС

15.Местые анестетики применяют в комбинации с адреналином, т.к.:

Выберите один ответ.

a.) ускоряется всасывание анестетика и усиливается местноанестезирующее действие.

B.) замедляется всасывание анестетика и усиливается местноанестезирующее действие;

C.) замедляется всасывание анестетика и ослабляется местноанестезирующее действие;

16.Метопролол показан для лечения:

Выберите один ответ.

a.) Атрио-вентрикулярной блокады

B.) Бронхиальной астмы

C.) Доброкачественной гиперплазии предстательной железы

D.) Артериальной гипертензии

17.Механизм действия вяжущих средств обусловлен:

Выберите один ответ.

a.) блоком натриевых каналов;

B.) адсорбцией химических соединений;

C.) покрытием слизистых оболочек пленкой, препятствующей раздражению чувствительных нервов. d.) коагуляцией белков и образованием пленки, предохраняющей окончания чувствительных нервов от раздражения;

18.Механизм действия местных анестетиков обусловлен:

Выберите один ответ.

a.) Блоком кальциевых каналов и удлинением абсолютного рефрактерного периода;

B.) Блоком калиевых каналов и невозможностью реполяризации мембраны;

C.) Активацией хлорных каналов и гиперполяризацией.

D.) Блоком натриевых каналов и невозможностью деполяризации мембраны;

19.Миорелаксант:

Выберите один ответ.

a.) скополамин.

B.) пипекуроний;

C.) атропин;

D.) пентамин;

20.Нежелательный побочный эффект ганглиоблокаторов:

Выберите один ответ.

a.) гипертензивный криз;

B.) повышение внутриглазного давления.

C.) ортостатический коллапс;

D.) бронхоспазм;

21.Неизбирательный бета-адреноблокатор:

Выберите один ответ.

a.) Метопролол

B.) Атенолол

C.) Празозин

D.) Пропранолол

22.Неостигмин (прозерин) используется для лечения миастении потому, что он улучшают холинергическую синаптическую передачу:

Выберите один ответ.

a.) В вегетативном ганглии

B.) В мионевральном соединении

C.) С постганглионарных холинергических волокон на клетки эффекторных органов

23.Норадреналин повышает:

Выберите один ответ.

a.) Периферическое сосудистое сопротивление

B.) Моторику желудочно-кишечного тракта

C.) Тонус бронхов

D.) Частоту сердечных сокращений

24.Почему атропин (третичный амин) превосходит метацин (четвертичное аммониевое соединение) в действии на ЦНС:

Выберите один ответ.

a.) лучше распределяется по организму (> значения Vd);

B.) лучше всасывается в системный кровоток из места введения (> коэффициент биодоступности);

C.) медленнее выводится (элиминируется) из организма (> значения Т1/2).

25.Почему галантамин (третичный амин) превосходит прозерин (четвертичное аммониевое соединение) в действии на ЦНС:

Выберите один ответ.

a.) Медленнее выводится (элиминируется) из организма (> значения Т1/2)

B.) Лучше всасывается в системный кровоток из места введения (> коэффициент биодоступности)

C.) Лучше распределяется по организму (> значения Vd)

26.Празозин вызывает:

Выберите один ответ.

a.) Снижение тонуса гладкой мускулатуры бронхов

B.) Снижение внутриглазного давления

C.) Урежение и ослабление сердечных сокращений

D.) Снижение периферического сосудистого сопротивления

27.Пропранолол вызывает:

Выберите один ответ.

a.) Снижение моторики желудочно-кишечного тракта

B.) Сокращение мышцы радужной оболочки глаза (мидриаз)

C.) Снижение тонуса бронхов

D.) Снижение частоты сердечных сокращений

28.Рефлекторную брадикардию вызывает:

Выберите один ответ.

a.) Сальбутамол

B.) Празозин

C.) Метопролол

D.) Карведилол

E.) Норадреналин

29.Сократительную активность миометрия снижает:

Выберите один ответ.

a.) Салбутамол

B.) Карведилол

C.) Пропранолол

D.) Норадреналин

E.) Метопролол

30.Средство для лечения острой сосудистой недостаточности:

Выберите один ответ.

a.) Метопролол

B.) Салбутамол

C.) Добутамин

D.) Норадреналин

E.) Пропранолол

31.Только для поверхностной анестезии используется:

Выберите один ответ.

a.) Бензокаин (анестезин).

B.) Бупивакаин;

C.) Прокаин (новокаин);

D.) Лидокаин;

32.Холиномиметики противопоказаны при:

Выберите один ответ.

a.) Миастении

B.) Болезни Альцгеймера

C.) Бронхиальной астме

D.) Ксеростомии

E.) Глаукоме

Предварительный просмотр:

Тема: «Химиотерапевтические средства»

Тестовые задания

1.В основе механизма действия АГ на синтез белка в микробных клетках лежит их способность угнетать:

Выберите один ответ.

a.) ДНК- полимеразу

B.) РНК-полимеразу

C.) процесс транспептидации

D.) процесс считывания кода м-РНК

2.Выбрать антибиотик – ингибитор синтеза белка в бактериальной клетке:

Выберите один ответ.

a.) бензилпенициллин

B.) карбенициллин

C.) ампициллин

D.) гентамицин

3.Выбрать определение «химиотерапии»

Выберите один ответ.

a.) химиотерапия – это подавление возбудителей заболеваний на поверхности тела человека (кожа, слизистые оболочки)

B) Химиотерапия – это подавление возбудителей заболеваний в окружающей среде (предметы ухода, инструменты, выделения больных)

C.) химиотерапия – это воздействие на клетки макроорганизма

D.) химиотерапия – это подавление возбудителей заболеваний во внутренних средах макроорганизма

4.К принципам химиотерапии относится следующие:

Выберите один ответ.

a.) все ответы правильны

B.) лечение антибактериальными препаратами следует начинать как можно раньше после начала заболевания

C.) препарат следует выбирать с учетом чувствительности возбудителя к химиотерапевтическому средству;

D.) дозу химиотерапевтического средства следует назначить с учетом тяжести заболевания у больного

5.К производным нитрофурана относится:

Выберите один ответ.

a.) Фталилсульфатиазол(фталазол)

B.) налидиксовая кислота

C.) фуразолидон

D.) нитрогексолин

6.Какие антибиотики, из перечисленных, нарушают синтез клеточной стенки:

Выберите один ответ.

a.) хлорамфеникол

B.) тетрациклины

C.) бета-лактамные антибиотики

D.) полимиксины

7.Какие препараты для лечения онихомикозов дают наименьшую частоту рецидивов?

Выберите один ответ.

a.) тербинафин и итраконазол;

B.) амфотерицин В и нистатин;

C.) гризеофульвин и леворин;

D.) цинка ундецилинат и йод

8.Какие препараты эффективны в отношении респираторно-синцитиальных вирусов и вирусов гриппа?

Выберите один ответ.

a.) рибавирин, интерферон;

B.) озельтамивир, ремантадин

C.) азидотимидин, саквинавир;

D.) ацикловир, фамцикловир;

9.Какое из утверждений отражает верно одни из общих принципов химиотерапии инфекционного заболевания.

Выберите один ответ.

B.) клиническое улучшение является основанием для прекращения терапии

C.) эффективность лечения часто не зависит от продолжительности антибактериальной терапии. d. 3) после клинического улучшения не следует прекращать лечение и продолжить по необходимости еще 48-72 часа.

10.Какой антибиотик эффективен при псевдомембранозном колите?

Выберите один ответ.

a.) диклоксацикллин

B.) фуразолидон

C.) ванкомицин

D.) ампициллин

11.Какой из перечисленных антибиотиков относится к бета-лактамным:

Выберите один ответ.

a.) стрептомицин

B.) меропенем

C.) тетрациклин

D.) полимиксин

12.Какой из химиотерапевтических препаратов относится к сульфаниламидам:

Выберите один ответ.

a.) менкомицин

B.) эритромицин

C.) стрептомицин

D.) сульфадимедин

13.Какой макролид имеет наименьший клиренс?

Выберите один ответ.

a.) эритромицин

B.) азитромицин

C.) кларитромицин

D.) рокситромицин

14.Какой пероральный препарат эффективен при грибковом менингите (например, криптококковом)?

Выберите один ответ.

a.) амфотерицин В;

B.) флуконазол

C.) тербинафин;

D.) кетоконазол;

15.Какой препарат бензилпенициллина относится к биосинтетическим:

Выберите один ответ.

a.) ампициллин

B.) бензилпенициллин-бензатин

C.) азлоциллин

D.) карбенициллин

16.Какой препарат применяется для лечения кандидозов ЖКТ?

Выберите один ответ.

a.) клотримазол

B.) гризеофульвин;

C.) нитрофунгин;

D.) нистатин;

17.Какой препарат применяется для лечения системных микозов?

Выберите один ответ.

a.) нистатин;

B.) клотримазол

C.) амфотерицин В;

D.) гризеофульвин;

18.Какой препарат эффективен в отношении вирусов гриппа типа А и В?

Выберите один ответ.

a.) ремантадин

B.) ацикловир;

C.) азидотимидин;

D.) озельтамивир;

19.Механизм действия сульфаниламидов связан с:

Выберите один ответ.

a) угнетением ЦОГ

B.) угнетением дигидрофолатредуктазы;

C.) конкурентным антагонизмом с ПАБК и угнетением дигидроптероатсинтетазы

D.) конкурентный антагонизм с ГАМК

20.Наиболее частым осложнением бета-лактамных антибиотиков являются

Выберите один ответ.

a.) аритмии

B.) угнетение кроветворения

C. 1) аллергические реакции

D.) снижение слуха

21.Полимиксины рассматриваются как препараты 3-го ряда («глубокого резерва») т.к.:

Выберите один ответ.

a.) обладают низкой эффективностью

B.) широко распространена к ним устойчивость

C.) ввиду низкой антимикробной активности

D.) ввиду высокой органотоксичности

22.При применении сульфаниламидов резорбтивного действия возможны следующие побочные эффекты:

Выберите один ответ.

a.) агранулоцитоз

B.) все вышеперечисленное

C.) кристалурия

D.) гемолитическая анемия, метгемоглобинемия

23.Противовирусные средства (ПВС) наиболее эффективны при раннем начале лечения, так как:

Выберите один ответ.

a.) ПВС проявляют виристатический эффект;

B.) ПВС проявляют вирицидный эффект;

C.) ПВС не проявляют органотоксичности

D.) ПВС органотоксичны;

24.Укажите антиретровирусные средства (для лечения ВИЧ-инфекции):

Выберите один ответ.

a.) арбидол, озельтамивир;

B.) азидотимидин, саквинавир;

C.) ацикловир, фамцикловир;

D.) интерферон, ганцикловир

25.Укажите механизм действия фторхинолонов:

Выберите один ответ.

a.) повышение проницаемости ЦПМ

B.) Угнетение синтеза бактериальной стенки

C.) угнетение ФДЭ-азы

D.) угнетение ДНК-гиразы

26.Укажите препарат относящийся к оксазолидинонам:

Выберите один ответ.

a.) линезолид

B.) моксифлоксацин

C.) ко-тримоксазол

D.) линкомицин

27.Укажите противогерпетическое средство:

Выберите один ответ.

a.) азидотимидин;

B.) ацикловир;

C.) арбидол;

D.) саквинавир

28.Что характерно для доксициклина?

Выберите один ответ.

a.) плохо всасывается из ЖКТ

B.) снижается биодоступность при приеме с пищей

C.) Т1/2 16-24 часа

D.) основной путь выведения через МВП

Предварительный просмотр:

Тема : «Общая фармакология»

Тестовые задания

1 . Вещества, обладающие аффинитетом и внутренней активностью, называют:

Выберите один ответ.

a.) антагонистами

B.) агонистами

2 . Действие веществ, развившееся после его поступления в системный кровоток, называется:

Выберите один ответ.

a.) резорбтивным

B.) местным

C.) побочным

D.) рефлекторным

3 . Как называется действие вещества, если оно взаимодействует только с функционально однозначными рецепторами определенной локализацией и не влияют на другие рецепторы?

Выберите один ответ.

a.) рефлекторное

B.) обратимое

C.) необратимое

D.) избирательное

4 . Как называется накопление в организме лекарственных веществ, при повторном его введении?

Выберите один ответ.

a.) тахифилаксия

B.) материальная кумуляция

C.) идиосинкразия

D.) сенсибилизация

5 . Как называется снижение эффективности действия вещества при повторном его введении?

Выберите один ответ.

a.) толерантность (привыкание)

B.) кумуляция

C.) идиосинкразия

D.) пристрастие

6.Как называется явление, когда отмена препарата вызывает психические и соматические нарушения, связанные с нарушениями функций многих систем организма вплоть до смертельного исхода?

Выберите один ответ.

a.) синдром отмены

B.) абстиненция

C.) сенсибилизация

D.) идиосинкразия

7.Какой из процессов протекает в фазу биотрансформации, которая называется конъюгацией?

Выберите один ответ.

a.) гидролиз

B.) восстановление

C.) кисление

D.) ацетилирование

8.Какой ответ наиболее соответствует термину «рецептор»?

Выберите один ответ.

a.) активные группировки макромолекул субстратов, с которыми лекарственное вещество взаимодействует

B.) транспортные системы, активированные лекарственным веществом

C.) ферменты окислительно-восстановительных реакций, активированные лекарством

D.) ионные каналы биологических мембран, проницаемость которых изменяет лекарственное вещество

9.Какой параметр фармакокинетики обозначается как «Т1/2»:

Выберите один ответ.

a.) константа скорости элиминации

B.) период полувыведения (полужизни, полуэлиминации) веществ

C.) абсорбция из места введения 50% вещества

D.) общий клиренс

10.Метаболическая биотрансформация это:

Выберите один ответ.

a.) взаимодействие с глюкуроновой кислотой

B.) превращение вещества за счет окисления, восстановления, гидролиза

C.) связывание с альбуминами плазмы крови

D.) метилирование и ацетилирование веществ

11.Объем распределения лекарств отражает:

Выберите один ответ.

a.) соотношение разовой и суточной доз лекарственного вещества

B.) гипотетический объем жидкости, в котором распределяется лекарство

C.) рассчитанное количество лекарства, достигшее системного кровотока

D.) соотношение дозы и массы тела

12.Объем распределения оказывается низким, если:

Выберите один ответ.

a.) вещество находится в плазме, в интерстициальной и внутриклеточной жидкости и накапливается в тканях

B.) вещество находится в плазме и в интерстициальной жидкости

C.) вещество находится в плазме, в интерстициальной и внутриклеточной жидкости

D.) вещество накапливается в плазме крови

13.Отметить основной механизм всасывания лекарственных веществ:

Выберите один ответ.

a.) пиноцитоз

B.) пассивная диффузия

C.) активные транспорт

D.) фильтрация

14.Фармакокинетика включает в себя:

Выберите один ответ.

a.) биотрансформацию лекарственных веществ в организме

B.) Влияние лекарств на генетический аппарат

C.) осложнения лекарственной терапии

D.) Влияние лекарств на обмен веществ в организме

15.Что включает в себя понятие фармакодинамика?

Выберите один ответ.

a.) метаболизм лекарственных веществ в организме

B.) условия хранения лекарственных средств

C.) биологические эффекты лекарственных средств

D.) способ введения лекарственных средств

16.Что входит в понятие «Биотрансформация»:

Выберите один ответ.

a.) связывание веществ с белками плазмы крови

B.) кумуляция веществ в жировой ткани

C.) комплекс физико-химических и биохимических превращений лекарственного вещества, направленных на выведение его из организма

D.) накопление лекарственного вещества в мышечной ткани

17.Что называется внутренней активностью вещества?

Выберите один ответ.

a.) способность вещества при взаимодействии с рецептором узнавать его

B.) способность вещества взаимодействовать с транспортными системами

C.) способность вещества при взаимодействии с рецептором стимулировать его и вызывать биологический эффект

D.) способность вещества взаимодействовать с белками плазмы

18.Что означает термин «аффинитет» ?

Выберите один ответ.

a.) сродство вещества к транспортным системам организма

B.) сродство вещества к альбуминам плазмы крови

C.) сродство лекарственных веществ к микросомальным ферментам печени

D.) сродство вещества к рецептору, приводящее к образованию с ним комплекса «вещество – рецептор»

19.Что означает термин «биодоступность»:

Выберите один ответ.

a.) степень связывания веществ с белками плазмы

B.) количество вещества в моче относительно исходной дозы препарата

C.) способность проходить через гематоэнцефалический барьер

D.) количество неизмененного вещества, которое достигло плазмы крови, относительно исходной дозы препарата

20.Что соответствует понятию «активные транспорт»:

Выберите один ответ.

a.) инвагинация клеточной мембраны с образованием вакуоли

B.) транспорт против градиента концентрации с затратой энергии

C.) транспорт по градиенту концентрации без затраты энергии

D. 1) облегченная диффузия

Предварительный просмотр:

Тема : «Иммунотропные средства»

Тестовые задания

1. Блокаторы H1-гистаминорецепторов применяются по всем следующим показаниям КРОМЕ:

Выберите один ответ.

a.) крапивница;

B.) бронхиальная астма

C.) лекарственная аллергия;

D.) сезонный ринит;

2.Какие виды фармакодинамических эффектов глюкокортикоидов находят применение в медицине?

Выберите один ответ.

a.) верны все, кроме 1

B.) гипергликемизирующее, подавление ростовых зон эпифизов;

C.) противошоковое, детоксицирующее (индукция ферментов печени);

D.) все указанное;

E.) иммунодепрессивное, противоаллергическое, противовоспалительное;

3.Какие препараты наиболее эффективны как противовоспалительные средства?

Выберите один ответ.

a.) антибиотики широкого спектра действия

B.) НПВС;

C.) СПВС;

D.) стабилизаторы мембран лаброцитов;

4.Каковы допустимые критерии эффективности инсулинотерапии?

Выберите один ответ.

a.) эугликемия, эуглюкозурия;

B.) эугликемия, аглюкозурия;

C.) агликемия, аглюкозурия

D.) нормогликемия, эуглюкозурия;

5.Каковы местные нежелательные эффекты при систематическом применении ГКС-содержащих мазей и кремов?

Выберите один ответ.

a.) отечность, гиперемия, болезненность;

B.) остеопороз, гирсутизм, дисменорея

C.) гипертрофия, гиперпигментация, кандидоз;

D.) повышение риска локальных инфекций, атрофия, депигментация;

6.Каковы наиболее опасные нежелательные эффекты системных ГКС при длительном применении?

Выберите один ответ.

a.) синдром отмены (недостаточность надпочечников);

B.) все указанное;

C.) синдром Иценко-Кушинга («кушингоид»);

D.) верны 1 и 2.

E.) иммунодефицитное состояние;

7.Каковы показания к применению препаратов гестагенов?

Выберите один ответ.

a.) заместительная гормонотерапия после овариогистерэктомии;

B.) рак молочной железы, рак простаты;

C.) дисфункциональные маточные кровотечения, привычный выкидыш, эндометриоз, контрацепция;

D.) контрацепция у женщин с высокой гипердислипидемией, сахарным диабетом, холестазом в анамнезе

8.Какое осложнение является наиболее частым при лечении препаратами инсулина?

Выберите один ответ.

a.) липодистрофия;

B.) гипокалиемия;

C.) инсулинорезистентность

D.) гипогликемия;

9.Какой препарат ГКС имеет низкую биодоступность при топическом (на кожу) применении?

Выберите один ответ.

a.) будесонид;

B.) флюоцинолона ацетонид (синафлан);

C.) преднизолона гемисукцинат

D.) бекламетазона пропионат;

10.Какой препарат ингаляционных ГКС даёт наименьший риск системных нежелательных эффектов?

Выберите один ответ.

a.) преднизолона гемисукцинат

B.) бекламетазона пропионат;

C.) будесонид;

D.) флюоцинолона ацетонид (синафлан);

11.Какой препарат относится к инсулиновым сенситайзерам?

Выберите один ответ.

a.) метформин;

B.) хумулин

C.) пиоглитазон;

D.) акарбоза;

E.) глибенкламид;

12.Какой препарат применяют при сезонных аллергических реакциях (поллинозах) только как средство профилактики?

Выберите один ответ.

a.) клемастин;

B.) гидрокортизон;

C.) кромогликат натрия;

D.) все указанное

13.Н1-гистаминоблокаторы II поколения отличаются от препаратов I поколения

Выберите один ответ.

a.) выраженным седативным действием;

B.) противорвотным действием

C.) значительным М-холиноблокирующим действием;

D.) большей избирательностью действия;

14.Окситоцин характеризуется всеми свойствами, КРОМЕ

Выберите один ответ.

a.) чувствительность матки постоянно высокая

B.) эффективен в малых дозах как родостимулятор;

C.) эффективен в больших дозах как утеротоник;

D.) чувствительность к нему матки возрастает к родам;

15.Показания к применению иммунодепрессантов-цитостатиков включают все, КРОМЕ:

Выберите один ответ.

a.) аутоиммунные болезни;

B.) профилактика РОТ

C.) тяжелые анафилактические реакции;

16.При гипотиреозе как средство заместительной терапии применяют

Выберите один ответ.

a.) протирелин;

B.) калия йодид;

C.) тиротропин

D.) левотироксин;

17. Применение иммунодепрессантов-цитостатиков (метотрексат, фторурацил, циклофосфамид) нередко осложняется

Выберите один ответ.

a.) лейкопенией и инфекционным синдромом;

B.) аллергией и фотодерматитом;

C.) кровоточивостью и анемией;

D.) сонливостью и заторможенностью

18. Тиамазол (мерказолил) как средство основного (длительного) лечения показан при…

Выберите один ответ.

a.) раке щитовидной железы;

B.) микседеме

C.) узловом токсическом зобе;

D.) диффузном токсическом зобе;

19.Указать верную последовательность применения препаратов при анафилактическом шоке:

Выберите один ответ.

a.) преднизолон – клемастин – аминофиллин – эпинефрин;

B.) клемастин (тавегил) - эпинефрин (адреналин) – преднизолон - аминофиллин (эуфиллин)

c.) эпинефрин – преднизолон – клемастин – аминофиллин

20.Чем купировать диабетическую кому?

Выберите один ответ.

a.) в/в 40-80 мл 40% раствора глюкозы;

B.) в/в 1 мл 0,1% раствора адреналина

C.) в/в 20 ЕД инсулин-цинк суспензия;

D.) в/в по 0.1ЕД/час инсулин короткого действия;

21.Что НЕ является абсолютным противопоказанием к применению препаратов эстрогенов?

Выберите один ответ.

a.) маточные кровотечения неясной природы;

B.) болезни печени, желтуха в анамнезе;

E.) тромбофилии;

Предварительный просмотр:

Тема : «Лекарственные средства влияющие

на функцию исполнительных органов»

Тестовые задания

1.Cредство для экстренного лечения гипертонического криза (при проявлении или нарастании признаков поражения “органов-мишеней”):

Выберите один ответ.

a.) метилдофа;

B.) каптоприл;

C.) нитропруссид натрия

D.) метопролол;

2.ААС для лечения наджелудочковых и желудочковых тахиаритмий:

Выберите один ответ.

a.) Верапамил

B.) Лидокаин

C.) Прокаинамид (Новокаинамид)

D.) Фенитоин (Дифенин)

3.ААС с минимальным потенциалом проаритмогенного действия:

Выберите один ответ.

a.) Пропранолол (Анаприлин)

B.) Амиодарон

C.) Пропафенон

D.) Лидокаин

4.ААС, применяющееся для лечения ИБС:

Выберите один ответ.

a.) Лидокаин

B.) Верапамил

C.) Хинидин

D.) Пропафенон

5.ААС, характеризующееся наибольшим периодом полувыведения:

Выберите один ответ.

a.) Хинидин

B.) Аденозин

C.) Лидокаин

D.) Амиодарон

6.Антигипертензивное нейротропное средство периферического действия:

Выберите один ответ.

a.) каптоприл;

B.) метопролол;

C.) нифедипин

D.) клофелин;

7.Антигипертензивное средство из группы блокаторов кальциевых каналов:

Выберите один ответ.

a.) нифедипин

B.) метопролол;

C.) каптоприл;

D.) лозартан;

8.Антигипертензивное средство из группы миотропных вазодилататоров:

Выберите один ответ.

a.) каптоприл;

B.) дилтиазем;

C.) дихлотиазид;

D.) метопролол

9.Антигипертензивное средство центрального действия:

Выберите один ответ.

a.) клофелин;

B.) пентамин

C.) натрия нитропруссид;

D.) каптоприл;

10.Антигипертензивное средство, блокирующее альфа- и бета-адренорецепторы: .

Выберите один ответ.

a.) карведилол;

B.) метопролол

C.) атенолол;

D.) пропранолол;

11.Антигипертензивное средство, для которого высоко вероятен риск развития эффекта первой дозы (резкая гипотензия в ортостатическом положении):

Выберите один ответ.

a.) метопролол;

B.) гидрохлортиазид;

C.) каптоприл;

D.) празозин

12.Антигипертензивное средство, противопоказанное при двустороннем стенозе почечных артерий:

Выберите один ответ.

a.) метопролол;

B.) верапамил;

C.) каптоприл;

D.) нифедипин

13.Антигипертензивное средство, снижающее образование ангиотензина II:

Выберите один ответ.

a.) верапамил;

B.) лозартан;

C.) каптоприл;

D.) празозин

14.Антигипертензивное средство, снижающее уровень ренина в крови:

Выберите один ответ.

a.) празозин;

B.) верапамил;

C.) пропранолол

D.) пентамин;

15.Антифибринолитическим действием обладает:

Выберите один ответ.

a.) фитоменадион

B.) хлористый кальций

C.) гепарин

D.) кислота аминокапроновая

16.Бета-адреноблокаторы применяют при ИБС в расчете на:

Выберите один ответ.

a.) снижение потребности в кислороде за счет снижения ЧСС и сократимости миокарда; b.) снижение потребности миокарда в кислороде за счет снижения преднагрузки;

C.) увеличение экстракции О2 из крови

D.) улучшение коронарного кровотока;

17.Веществом, активирующим фибринолиз является:

Выберите один ответ.

a.) Варфарин

B.) клопидогрел

C.) гирудин

D.) стрептокиназа

18.Все кардиотонические средства повышают:

Выберите один ответ.

a.) атрио-вентрикулярную проводимость;

B.) сократимость миокарда;

C.) автоматизм синоатриального узла

D.) кислородный запрос миокарда;

19.Главный элемент антигипертензивного действия альфа-адреноблокаторов:

Выберите один ответ.

a.) Венозная вазодилатация;

B.) Отрицательное хроно- и инотропное действие

C.) Артериолярная вазодилатация;

20.Главный элемент антигипертензивного действия бета-адреноблокаторов:

Выберите один ответ.

a.) Отрицательное хроно- и инотропное действие

B.) Артериолярная вазодилатация;

C.) Венозная вазодилатация;

D.) Блокада рецепторов ангиотензина II;

21.Для лечения артериальной гипертензии НЕ применяется:

Выберите один ответ.

a.) фуросемид

B.) спиронолактон;

C.) гидрохлортиазид;

D.) маннит;

22.Для систематического лечения артериальной гипертензии НЕ применяется:

Выберите один ответ.

a.) метопролол;

B.) лозартан;

C.) фентоламин

D.) нифедипин;

23.К бронхолитикам из группы симпатомиметиков относится:

Выберите один ответ.

a.) изадрин

B.) эфедрин

C.) салбутамол

24.К негликозидным кардиотоникам относятся все препараты, КРОМЕ:

Выберите один ответ.

a.) милринона

B.) строфантина (уабаина);

C.) допамина;

D.) добутамина;

25.К препаратам антикоагулянтов непрямого действия относится:

Выберите один ответ.

a.) гирудин

B.) Гидроцитрат натрия

C.) фраксипарин

D.) варфарин

26.К сердечным гликозидам (СГ) относятся все нижеперечисленные препараты, КРОМЕ:

Выберите один ответ.

a.) дигоксина;

B.) добутамина;

C.) дигитоксина;

D.) строфантина

27.Какая комбинация диуретиков рациональна?

Выберите один ответ.

a.) Фуросемид + маннит

B.) Маннит + мочевина

C.) Дихлотиазид + триамтерен

D.) Фуросемид + кислота этакриновая

28.Какие препараты нитроглицерина больные стенокардией применяют для купирования приступов?

Выберите один ответ.

a.) нитроглицерин в мази

B.) нитроглицерин в сублингвальных таблетках;

C.) нитроглицерин в микродраже (сустак);

D.) нитроглицерин в растворе для в/в введения;

29.Какие признаки передозировки СГ являются жизнеугрожающими?

Выберите один ответ.

a.) утомляемость, мышечная слабость

B.) диспептические расстройства;

C.) зрительные расстройства;

D.) желудочковые тахиаритмии;

30.Какие средства применяют для лечения ИБС?

Выберите один ответ.

a.) все указанное

B.) антиатеросклеротические средства;

C.) антитромботические средства;

D.) кардиопротекторные средства;

31.Каким средством можно купировать спазмы кишечника (колики)?

Выберите один ответ.

a.) метамизол (анальгин);

B.) метоклопрамид;

C.) дротаверин (но-шпа).

D.) морфин;

E.) магния сульфат;

32.Какое из антацидных средств может вызвать алкалоз при систематическом применении?

Выберите один ответ.

a.) магния трисиликат;

B.) алюминия гидроокись;

C.) кальция глюконат;

D.) натрия гидрокарбонат

E.) магния окись;

33.Какое противорвотное средство применяют при рефлюксах, парезе желудка?

Выберите один ответ.

a.) ондансетрон (зофран);

B.) хлорпромазин (аминазин);

C.) метоклопрамид;

D.) дифенгидрамин (димедрол);

E.) перфеназина гидрохлорид (этаперазин)

34.Какой диуретик может вызвать снижение слуха?

Выберите один ответ.

a.) спиронолактон

B.) Дихлотиазид

C.) маннит

D.) фуросемид

35.Какой препарат используется для усиления сократительной активности миометрия в родах?

Выберите один ответ.

a.) эргометрина малеат

B.) Атропина сульфат

C.) окситоцин

D.) папаверин

36.Какой препарат используют для остановки маточных кровотечений?:

Выберите один ответ.

a.) эргометрина малеат

B.) Атропина сульфат

C.) фенотерол

D.) простагландин F-2а

37.Какой препарат относится к антикоагулянтам прямого действия?

Выберите один ответ.

a.) фибринолизин

B.) фитоменадион

C.) Гепарин

D.) Варфарин

38.Какой препарат применяется только для профилактики рвоты, вызванной укачиванием (морская болезнь)?

Выберите один ответ.

a.) метоклопрамид (церукал);

B.) перфеназина гидрохлорид (этаперазин)

C.) дипразин (пипольфен);

D.) «Аэрон»;

E.) ондансетрон (зофран);

39.Какой препарат снижает сократимость миометрия?

Выберите один ответ.

a.) фенотерол

B.) питуитрин

C.) простагландин F-2а

D.) Папаверин

40.Механизм бронхолитического действия атропина связан с:

Выберите один ответ.

a.) прямым миотропным действием на гладкие мышцы бронхов

B.) блокадой м-холинорецепторов гладких мышц бронхов

C.) возбуждением В2-адренорецепторов

41.Механизм действия петлевых диуретиков(фуросемид и т.д.):

Выберите один ответ.

a.) Повышают осмотическое давление жидкости в просвете канальцев

B.) Снижают реабсорбцию натрия, хлора и калия в толстой части восходящего колена петли Генли

c.) Увеличивают клубочковую фильтрацию

D.) Блокируют карбангидразу

42.Механизм действия тиазидных диуретиков?

Выберите один ответ.

a.) Повышают осмотическое давление жидкости в канальцах нефрона

B.) Увеличивают клубочковую фильтрацию

C.) Блокируют рецепторы альдостерона

d.) Снижают реабсорбцию натрия, хлора в дистальных канальцах

43.Механизм отхаркивающего действия препаратов термопсиса осуществляется за счет:

Выберите один ответ.

a.) раздражения рецепторов желудка и рефлекторного усиления секреции бронхиальных желез

b.) прямой стимуляции секреции бронхиальных желез

c.) разжижения мокроты при деполимеризации белков

44.Наиболее подходящим показанием для назначения СГ является:

Выберите один ответ.

a.) нестабильная стенокардия;

b.) ХЗСН с выраженной брадикардией;

c.) ХЗСН с множественными желудочковыми экстрасистолами

d.) ХЗСН с мерцательной аритмией предсердий;

45.Общее свойство всех ААС (кроме сердечных гликозидов), применяемых для лечения тахиаритмий:

Выберите один ответ.

a.) Замедление быстрой деполяризации

b.) Замедление реполяризации

c.) Ускорение реполяризации

d.) Снижение автоматизма

46.Основным свойством гепарина является:

Выберите один ответ.

a.) Кумулирует

b.) эффективен при приеме внутрь

c.) Действие развивается через 18-24 часа

d.) Задерживает свертывание крови «in vivo» и «in vitro»

47.Особенность действия лидокаина:

Выберите один ответ.

a.) Замедляет быструю деполяризацию

b.) Ускоряет реполяризацию

c.) Замедляет проводимость

d.) Повышает АД

48.Отметить антиагрегант - ингибитор ЦОГ:

Выберите один ответ.

a.) варфарин

b.) фитоменадион

c.) ацетилсалицилова кислота

d.) Гидроцитрат натрия

49.Отметить коагулянт прямого действия:

Выберите один ответ.

a.) фитоменадион

b.) тромбин

c.) апротинин

d.) Гепарин

50.Отметить побочное действие эуфилина:

Выберите один ответ.

a.) угнетение дыхания

b.) повышение потребности миокарда в кислороде

c.) повышение АД

51.Отметить препарат относящийся к гидроуретикам:

Выберите один ответ.

a. 2) индапамид

b. 3) маннит

c. 1) Дихлотиазид

d. 4) фуросемид

52.Отметить препарат относящийся к салуретикам:

Выберите один ответ.

a.) Мочевина

b.) Маннит

c.) Демеклоциклин

d.) Фуросемид

53.Отметить препарат, усиливающий отток желчи (холекинетик):

Выберите один ответ.

a.) «Холензим»;

b.) кислота дегидрохолевая;

c.) дротаверин (но-шпа);

d.) магния сульфат;

e.) атропин;

f.) аминофиллин (эуфиллин)

54.Отметить препарат-холесекретик растительного происхождения:

Выберите один ответ.

a.) Магния сульфат;

b.) осалмид (оксафенамид);

c.) «Аллохол»;

d.) «Холензим»

e.) дротаверин (но-шпа);

55.Отметить слабительное для экстренного очищения кишечника (подготовка к лечебным или диагностическим процедурам):

Выберите один ответ.

a.) лактулоза;

b.) магния сульфат;

c.) изафенин;

d.) свечи глицериновые;

e.) фенолфталеин

56.Отметить средство заместительной терапии при хроническом панкреатите:

Выберите один ответ.

a.) контрикал;

b.) пентагастрин

c.) мизопростол;

d.) атропин;

e.) панкреатин;

57.Отметить средство, понижающее секрецию соляной кислоты в желудке:

Выберите один ответ.

a.) омепразол;

b.) натрия гидрокарбонат;

c.) алюминия гидроокись

d.) гистамин;

e.) пентагастрин

58.Почему коронаролитики (например, дипиридамол) могут вызвать «феномен обкрадывания» миокарда?

Выберите один ответ.

a.) тонизируют коронарные сосуды;

b.) увеличивают сократимость миокарда

c.) перераспределяют кровоток в здоровые сосуды в ущерб ишемизированной зоне миокарда;

d.) расширяют сосуды большого круга кровообращения;

59.Препаратом, снижающим реактивность бронхов, из группы ГК при бронхиальной астме применяют:

Выберите один ответ.

a.) бекламетазона дипропионат

b.) кромолин натрия

c.) ипратропия бромид

60.Препаратом выбора для купирования бронхоспазма является:

Выберите один ответ.

a.) изадрин

b.) сальбутамол

c.) атропин

61.При каких заболеваниях применяют диуретики для плановой терапии:

Выберите один ответ.

a.) острые отравления

b.) Отек мозга

c.) гипертоническая болезнь

d.) Отек легких

62.При отеке легких для снижения давления в малом круге кровообращения применяют:

Выберите один ответ.

a.) ганглиоблокаторы

b.) ингаляции кислорода

c.) стимуляторы дыхания

63.При отеке легких используют ингаляции раствора этилового спирта с целью:

Выберите один ответ.

a.) противовспенивающего действия

b.) наркотического эффекта

c.) дегидратирующего эффекта

64.Противоаритмическое действие сердечных гликозидов обусловлено:

Выберите один ответ.

a.) Снижением силы сердечных сокращений

b.) Замедлением проводимости

c.) Снижением автоматизма

d.) Снижением возбудимости

65.Противокашлевым средством, подавляющим кашлевой рефлекс, блокирующим возбудимость чувствительных окончаний в дыхательных путях, является:

Выберите один ответ.

a.) тусупрекс

b.) кодеин

c.) либексин

66.Смешанное стимулирующие действие на дыхательный центр оказывает:

Выберите один ответ.

a.) кофеин

b.) никетамид (кордиамин)

c.) цититон

67.Специфический нежелательный побочный эффект ингибиторов ангиотензинпревращающего фермента:

Выберите один ответ.

a.) сухой кашель

b.) агранулоцитоз;

c.) ринорея;

d.) анорексия;

68.Средство для лечения брадиаритмий

Выберите один ответ.

a.) Верапамил

c.) Лидокаин

d.) Атропин

69.Средство для лечения только желудочковых тахиаритмий

Выберите один ответ.

a.) Пропафенон

b.) Прокаинамид (Новокаинамид)

c.) Лидокаин

d.) Верапамил

70.Средство для лечения только суправентрикулярных тахиаритмий

Выберите один ответ.

a.) Лидокаин

b.) Прокаинамид (Новокаинамид)

c.) Верапамил

d.) Пропафенон

71.Укажите наиболее частый нежелательный эффект нитратов:

Выберите один ответ.

a.) метгемоглобинобразование;

b.) головная боль;

c.) снижение тонуса ЖВП

d.) подавление агрегации тромбоцитов;

72.Что является показанием для назначения метоклопрамида?

Выберите один ответ.

a.) понос;

b.) пониженная кислотность;

c.) повышенная кислотность;

d.) кинетозы (морская, воздушная болезнь);

e.) тошнота, рвота.

Предварительный просмотр:

Тема: «Лекарственные средства влияющие

на центральную нервную систему»

Тестовые задания

1.Что верно об АСК?

Выберите один ответ.

a.) не применяют при артритах;

b.) наименее ульцерогенна;

c.) не применяют у детей до 14 лет при лихорадке;

d.) как антиагрегант применяют в больших, нежели анальгетические, дозах

2.Что НЕ относится к противопоказаниям для применения опиоидных анальгетиков?

Выберите один ответ.

a.)угнетение дыхания;

b.)инфаркт миокарда;

c.)черепно-мозговые травмы

d.)острые боли в животе неясного генеза;

3.Что применяют для восстановления дыхания при передозировке героина (морфина)?

Выберите один ответ.

a.)налоксон;

b.)кислород;

c.)трамадол;

d.)налтрексон

4.Что характерно для жаропонижающего эффекта ненаркотических анальгетиков?

Выберите один ответ.

a.) НА вызывают гипотермию, подавляя теплопродукцию;

b.) назначение обязательно при субфебрильной температуре;

c.) НА снижают лихорадку, усиливая теплоотдачу

d.) он присущ всем НА в дозах, существенно превышающих анальгетические;

5.Что характерно для производных пиразолона (метамизол (анальгин), фенилбутазон (бутадион))?

Выберите один ответ.

a.) применяют как антиагреганты при ИБС

b.) применяют для длительной терапии артритов;

c.) гематотоксичны;

d.) нет противовоспалительного эффекта;

6.Что характерно для противовоспалительного действия НПВС?

Выберите один ответ.

a.) улучшают качество жизни больных, временно уменьшая симптомы артрита;

b.) излечивают артриты при полноценном курсовом лечении;

c.) угнетают все фазы воспаления;

d.) противовоспалительный эффект обусловлен ингибированием синтеза лейкотриенов 7.Что является причиной смерти при передозировке опиоидных анальгетиков?

Выберите один ответ.

a.)бронхоспазм;

b.)отек легких;

c.)остановка дыхания;

d.)остановка сердца

8.Экстрапирамидные двигательные нарушения - типичный нежелательный побочный эффект:

Выберите один ответ.

a.) Клозапина

b.) Галоперидола

c.) Оланзапина

d.) Рисперидона

9.Какой препарат применяют при эпилептическом статусе?

Выберите один ответ.

a.)дифенгидрамин (димедрол);

b.)диазепам;

c.)этосуксимид

10.Какой эффект опиоидных анальгетиков может быть опасен даже при однократном применении на фоне сильной боли

Выберите один ответ.

a.)спазмы в ЖКТ;

b.)эйфория;

c.)запор

d.)угнетение дыхания;

11.Какой эффект опиоидных анальгетиков ограничивает их широкое применение?

Выберите один ответ.

a.)анальгетический;

b.)седативный;

c. 3)эйфоризирующий;

d. 4)спазмогенный

12.Кофеин:

Выберите один ответ.

a.) Тонизирует дыхательный и сосудодвигательный центры

b.) Суживает коронарные сосуды

c.) Расширяет мозговые сосуды

13.Моклобемид по сравнению с имипрамином оказывает более сильное:

Выберите один ответ.

a.) Психостимулирующее действие

b.) Психоседативное действие

c.) Альфа-адреноблокирующее действие

d.) М-холиноблокирующее действие

14.Нимесулид и целекоксиб - селективные ингибиторы ЦОГ-2 - отличаются от неселективных (АСК, диклофенака и др.):

Выберите один ответ.

a.) большей эффективностью;

b.) меньшей частотой гастропатии;

c.) меньшей аллергенностью;

d.) существенно меньшей выраженностью всех «ПГ-зависимых» побочных эффектов

15.Основное показание для применения опиоидных (наркотических) анальгетиков

Выберите один ответ.

a.)травматические и висцеральные боли высокой интенсивности

b.)травматические и висцеральные боли умеренной интенсивности;

c.)невралгия;

d.)остеоалгия;

16.Отметить общие для НА/НПВС («ЦОГ- и ПГ-зависимые») нежелательные эффекты:

Выберите один ответ.

a.) привыкание, лекарственная зависимость;

b.) онливость, угнетение дыхания;

c.) аллергические реакции, лейкопения

d.) гастропатии, кровоточивость;

17.Отметить правильное утверждение о кеторолаке:

Выберите один ответ.

a.) эффективен только при умеренных болях;

b.) применяется для длительной терапии артритов;

c.) применяется не долее 5-7 дней из-за нефротоксичности

d.) гепатотоксичен;

18.При паркинсонизме применяют:

Выберите один ответ.

a.)фенитоин (дифенин);

b.)карбамазепин;

c.)леводопа

19.Противорвотное действие оказывает:

Выберите один ответ.

a.) Хлорпромазин

b.) Буспирон

c.) Зопиклон

d.) Диазепам

20.Противосудорожное действие оказывает:

Выберите один ответ.

a.) Галоперидол

b.) Диазепам

c.) Буспирон

d.) Хлорпромазин

21.Психостимулятор из группы метилксантинов:

Выберите один ответ.

a.) Амфетамин

b.) Кофеин

c.) Моклобемид

d.) Пирацетам

e.) Имипрамин

22.Специфическое средство лечения острого отравления транквилизаторами производными бензодиазепина:

Выберите один ответ.

a.) Зопиклон

b.) Флумазенил

c.) Кофеин

d.) Пирацетам

e.) Феназепам

23.Транквилизатор с самым длительным периодом полувыведения (Т1/2 >48 часов):

Выберите один ответ.

a.) Диазепам

b.) Оксазепам

c.) Лоразепам

d.) Медазепам

e.) Мидазолам

24.Трициклический антидепрессант:

Выберите один ответ.

a.) Кофеин

b.) Амитриптилин

c.) Флуоксетин

d.) Пирацетам

25.Указать особенности обезболивающего эффекта НА:

Выберите один ответ.

a.) эффективны при умеренных артралгиях, миалгиях, цефалгиях;

b.) устраняют боли любой интенсивности;

c.) эффективнее наркотических анальгетиков при сильных травматических и висцеральных болях;

d.) при длительном приеме развивается толерантность

26.Флуоксетин по сравнению с амитриптилином::

Выберите один ответ.

a.) Менее токсичен

b.) Оказывает более сильное М-холиноблокирующее действие

c.) Характеризуется большей клинической эффективностью

d.) Оказывает более сильное седативное действие

27.Чем отличаются снотворные производные барбитуровой кислоты от производных бензодиазепина?

Выберите один ответ.

a.)выраженное центральное миорелаксирующие действие

b.)больше нарушение структуры сна;

c.)слабая индукция микросомальных ферментов печени;

28.Чем парциальные агонисты и агонисты-антагонисты опиоидных рецепторов (пентазоцин, бупренорфин) отличаются от полных агонистов (морфин)?

Выберите один ответ.

a.)сильнее спазмогенное действие;

b.)слабее наркогенность;

c.)возможно ректальное введение

d.)сильнее угнетение дыхания;

29.Аллостерический активатор ГАМК-А рецепторов:

Выберите один ответ.

a.) Баклофен

b.) Диазепам

c.) Буспирон

d.) Амизил

30.Антидепрессант избирательный ингибитор МАО-А:

Выберите один ответ.

a.) Моклобемид

b.) Пирацетам

c.) Флуоксетин

d.) Имипрамин

e.) Амитриптилин

f.) Кофеин

31.Антидепрессант селективный ингибитор обратного захвата серотонина:

Выберите один ответ.

a.) Пирацетам

b.) Флуоксетин

c.) Кофеин

d.) Имипрамин

e.) Амитриптилин

32.Антиманиакальное действие не оказывает:

Выберите один ответ.

a.) Галоперидол

b.) Лития карбонат

c.) Диазепам

d.) Трифтазин

33.Антипсихотическое средство из группы производных фенотиазина:

Выберите один ответ.

a.) Рисперидон

b.) Оланзапин

c.) Хлорпромазин

d.) Клозапин

e.) Галоперидол

34.Атипичный нейролептик:

Выберите один ответ.

a.) Фторфеназин

b.) Галоперидол

c.) Клозапин

d.) Хлорпромазин

e.) Трифтазин

35.Главный психотропный эффект пирацетама:

Выберите один ответ.

a.) Анксиолитический

b.) Седативный

c.) Мнемотропный

d.) Психостимулирующий

36.Для НПВС характерны все нижеприведенные лекарственные взаимодействия, КРОМЕ:

Выберите один ответ.

a.) кодеин ослабляет обезболивающее действие НА или НПВС;

b.) НПВС ослабляют эффект диуретиков и ряда антигипертензивных средств;

c.) алюминий-содержащие антациды снижают биодоступность НПВС

d.) седативные средства усиливают анальгезирующий эффект НПВС;

37.Дневной транквилизатор:

Выберите один ответ.

a.) Феназепам

b.) Зопиклон

c.) Медазепам

d.) Диазепам

e.) Аминазин

38.Желудочковые тахиаритмии потенциальный нежелательный побочный эффект:

Выберите один ответ.

a.) Типичных нейролептиков

b.) Трициклических антидепрессантов

c.) Антидепрессантов селективных ингибиторов обратного захвата серотонина

d.) Транквилизаторов производных бензодиазепина

e.) Атипичных нейролептиков

39.К симптомам острого отравления снотворными относится:

Выберите один ответ.

a.)возбуждение, повышение АД;

b.)коматозное состояние, угнетения дыхания, гипоксия;

c.)повышение температуры, усиление рефлекторной возбудимости

40.Какие опиоидные анальгетики противопоказаны при инфаркте миокарда?

Выберите один ответ.

a.)пентазоцин, буторфанол;

b.)морфин, промедол;

c.)фентанил, налбуфин

41.Какие препараты можно использовать при спастичности скелетных мышц?

Выберите один ответ.

a.)стрихнин, никетамид (кордиамин), бемегрид

b.)баклофен, диазепам, мидокалм;

c.)прозерин,галантамин, физостигмин;

42.Какое утверждение в отношении ацетаминофена (парацетамола) НЕ верно?

Выберите один ответ.

a.) НПВС выбора при артритах

b.) гастротоксичен;

c.) отсутствует антиагрегантный эффект;

d.) жаропонижающее средство выбора при вирусных инфекциях у детей;

43.Какой высокоактивный опиоидный анальгетик предпочтителен для обезболивания при краткосрочных болезненных манипуляциях/операциях?

Выберите один ответ.

a.)морфин;

b.)пентазоцин

c.)фентанил;

d.)промедол;

44. Какой препарат относится к противоэпилептическим средствам?

Выберите один ответ.

a.)натрия вальпроат;

b.)леводопа;

c.)циклодол

45.Какой препарат относится к снотворным средствам?

Выберите один ответ.

a.)циклодол;

b.)зопиклон;

c.)фенитоин (дифенин);

d.)леводопа

46.Какой препарат предпочтителен для обезболивания I периода родов?

Выберите один ответ .

a.)кодеин

b.)метамизол (анальгин);

c.)морфин;

d.)тримеперидин (промедол);

a.) препаратами меди

b.) фосфором

c.) соединениями ртути

d.) соединениями железа

2.Для какого лекарственного вещества первичная фармакологическая реакция обусловлена влиянием на процессы транскрипции ДНК:

Выберите один ответ.

a.) инсулин;

b.) бензилпенициллин

c.) гепарин;

d.) преднизолон;

3.Для какого лекарственного вещества первичная фармакологическая реакция обусловлена снижением проницаемости потенциалзависимых ионных каналов:

Выберите один ответ.

a.) дигитоксин;

b.) лидокаин;

c.) ропин;

d.) фуросемид

4.Для какого лекарственного вещества первичная фармакологическая реакция обусловлена снижением проницаемости медиаторзависимых (хемочувствительных) ионных каналов:

Выберите один ответ.

a.) лидокаин;

b.) пипекуроний

c.) парацетамол;

d.) верапамил;

5.Для какого лекарственного вещества первичная фармакологическая реакция обусловлена угнетением активности фермента:

Выберите один ответ.

a.) лидокаин;

b.) адреналин;

c.) прозерин

d.) атропин;

6.Для какого лекарственного вещества первичная фармакологическая реакция обусловлена угнетением процесса облегченной диффузии:

Выберите один ответ.

a.) адреналин;

b.) дихлотиазид.

c.) дигоксин;

d.) диазепам;

7.Для снижения концентрации яда в крови и тканях используют:

Выберите один ответ.

a.) слабительные средства

b.) химические антидоты

c.) адсорбенты

d.) функциональные антидоты

8.Для удаления невсосавшегося яда из желудка производят промывание последнего водой с добавлением:

Выберите один ответ.

a.) раствора атропина

b.) сульфата натрия

c.) метилтиониния хлорида (метиленового синего)

d.) активированного угля

9.К комплексонам относится:

Выберите один ответ.

a.) пентацин

b.) налоксон

c.) натрия тиосульфат

d.) пентамин

10.Какие средства используются для стимуляции дыхательного центра:

Выберите один ответ.

a.) никетамид (кордиамин); бемегрид; сульфокамфокаин;

b.) морфин; фентанил; тримеперидин (промедол)

c.) эпинефрин (адреналин); фенилэфрин (мезатон); норэпинефрин (норадреналин)

d.) дротоверин (но-шпа); метацин; папаверин;

11.Натрия тиосульфат образует малотоксичные роданистые соединения при отравлении:

Выберите один ответ.

a.) героином

b.) цианидами

c.) атропином

d.) сердечными гликозидами

12.Основными целями лечения острых отравлений являются все кроме:

Выберите один ответ.

a.) снижение концентрации яда в крови и тканях

b.) уменьшение дальнейшего всасывания яда

c.) нормализация функций жизненно важных органов и систем

d.) замедление метаболизма яда

13.Функциональным антидотом морфина является:

Выберите один ответ.

a.) дифенгидрамин (димедрол)

b.) атропин

c.) налоксон

d.) бемегрид

14.Химическим антидотом при передозировке Гепарина является:

Выберите один ответ.

a.) фитоменадион

b.) хлорид кальция

c.) протамина сульфат

d.) димеркапрол (унитиол)

15.Этиловый спирт изменяет метаболизм яда при отравлении:

Выберите один ответ.

a.) метиловым спиртом

b.) атропином

c.) морфином

d.) препаратами мышьяка


1.Как называется раздел фармакологии, изучающий всасывание, распределение, биотрансформацию и выведение лекарственных средств?

Фармакокинетика.

Фармакодинамика.

2.Как называется раздел фармакологии, изучающий виды действия лекарственных средств, фармакологические эффекты, механизм действия?

Фармакодинамика.

Фармакокинетика.

3.Основной механизм всасывания лекарственных средств в желудочно-кишечном тракте:

Активный транспорт.

Облегчённая диффузия.

Пассивная диффузия через мембраны клеток.

Пиноцитоз.

4.Основное место всасывания лекарств – слабых оснований:

Тонкий кишечник.

5.Основное место всасывания лекарств – слабых кислот:

Тонкий кишечник.

6.Какой способ введения лекарственных средств обеспечивает 100 %-ю биодоступность?

Внутримышечный.

Ректальный.

Внутривенный.

Через рот.

7.Как изменится всасывание лекарственных средств – слабых кислот при уменьшении кислотности желудочного сока?

Увеличится.

Уменьшится.

8.Как изменится всасывание лекарственных средств – слабых оснований при уменьшении кислотности желудочного сока?

Увеличится.

Уменьшится.

9.Путём пассивной диффузии через биологические мембраны легко транспортируются вещества:

Липофильные.

Полярные.

Гидрофильные.

10.Энтеральный путь введения лекарственных средств:

Внутримышечный.

Ингаляционный.

Сублингвальный.

Внутривенный.

11.Парентеральный путь введения лекарственных средств:

Через рот.

В прямую кишку.

Подкожный.

Сублингвальный.

12.Где в основном происходят, всасывание большей части лекарственных средств?

В ротовой полости.

В желудке.

В тонком кишечнике.

В толстом кишечнике.

13.Внутривенно можно вводить:

Масляные растворы.

Нерастворимые соединения.

Осмотически активные соединения.

Микрокристаллические взвеси.

Нерастворимые соединения.

14.Какое функциональное изменение в организме вызывают сердечные гликозиды при сердечной недостаточности?

Возбуждение.

Угнетение.

Тонизирование.

Успокоение.

15.Какое функциональное изменение в организме вызывает средство, понижающее артериальное давление при артериальной гипертензии?

Возбуждение.

Угнетение.

Тонизирование.

Успокоение.

16.Как называется накопление в организме лекарственного средства при его повторных введениях?

Функциональная кумуляция.

Сенсибилизация.

Материальная кумуляция.

Тахифилаксия.

17.Толерантность – это:

Аллергическая реакция организма на повторное введение лекарства.

Уменьшение фармакологического эффекта на повторное введение лекарства.

Непреодолимое стремление к повторному приёму лекарства.

18.Снижение эффекта при введении лекарственных средств через короткие промежутки времени – это:

Тахифилаксия.

Идиосинкразия.

Сенсибилизация.

Пристрастие.

19.Побочный эффект, который может возникнуть только при повторных введениях лекарственных средств:

Идиосинкразия.

Тератогенное действие.

Мутагенное действие.

Привыкание.

20.Побочный эффект, который может возникнуть только при применении психотропных средств:

Идиосинкразия.

Пристрастие.

Привыкание.

Сенсибилизация.

21.Определите вид взаимодействия лекарственных средств: больному при отравлении мускарином проведено промывание желудка взвесью активированного угля:

Суммированный синергизм.

Химический антагонизм.

Конкурентный антагонизм.

Физический антагонизм.

22.Мутагенное действие – это:

23.Тератогенное действие – это:

Повреждение генетического аппарата зародышевой клетки.

Нарушение дифференцировки тканей плода, вызывающее различные аномалии.

Побочный эффект, возникающий в первые 12 недель после оплодотворения и вызывающий гибель зародыша.

24.Эмбриотоксическое действие – это:

Повреждение генетического аппарата зародышевой клетки.

Нарушение дифференцировки тканей плода, вызывающее различные аномалии.

Побочный эффект, возникающий в первые 12 недель после оплодотворения и вызывающий гибель зародыша.

КЛИНИЧескаЯ фармакологиЯ

001. Как меняется объем распределения жирорастворимых ЛС

у тучных больных?

а) уменьшается

в) не меняется

г) не меняется или увеличивается

д) увеличивается

002. Как меняется биотрансформация ЛС на фоне курения и приема алкоголя?

а) уменьшается

б) уменьшается или не меняется

в) не меняется

г) не меняется или усиливается

д) усиливается

003. От какого основного фактора зависит биоэквивалентность лекарства?

а) фармакодинамические характеристики ЛС

б) физико-химические характеристики

в) лекарственная форма

г) технология изготовления

д) состояние организма пациента

004. Какие побочные действия ЛС не зависят от дозы?

а) связанные с фармакологическими свойствами ЛС

б) токсические осложнения,

обусловленные абсолютной или относительной передозировкой

в) вторичные эффекты, обусловленные нарушением

иммунобиологических свойств организма

г) иммунологические реакции немедленного и замедленного типов

д) синдром отмены

005. Какое осложнение у новорожденных

может вызвать введение беременным перед родами сульфата магния?

а) развитие нейро-мышечных блокад и летаргии

б) угнетение дыхания

в) тромбоцитопению

г) гепатотоксическое действие

д) гипотрофию

006. Какие осложнения у новорожденных

может вызвать назначение беременным -адреноблокаторов?

а) потеря слуха

б) поражение кожных покровов

в) гипотрофию плаценты и плода

г) преждевременное закрытие боталлова протока

д) геморрагический синдром

007. Применение каких антимикробных препаратов

наиболее безопасно во время беременности?

а) аминогликозиды

б) котримоксазол

в) нитрофураны

г) пенициллины

д) фторхинолоны

008. Укажите антибиотик, не обладающий антисинегнойной активностью:

а) карбенициллин

б) ампиокс

в) тикарциллин

д) азлоциллин

г) цефтазидим

009. Больной пневмонией, получающий антибактериальное лечение,

стал предъявлять жалобы на головокружение,

неустойчивость и пошатывание при ходьбе.

Применение какого антибиотика могло вызвать эти симптомы?

а) ампициллин

б) цефоперазон

в) гентамицин

г) эритромицин

д) линкомицин

010. Укажите антибиотик, являющийся препаратом выбора

при лечении инфекций, вызванных staph. aureus:

а) пенициллин

б) гентамицин

в) азитромицин

г) левомицетин

д) амоксициллин-клавуланат

011. Укажите антибиотик, являющийся препаратом выбора

при лечении инфекций,

вызванных метициллинрезистентным стафилококком:

а) линкомицин

б) эритромицин

в) ванкомицин

г) пенициллин

д) оксациллин

012. Укажите антибиотик,

а) ампициллин

б) гентамицин

в) цефоперазон

г) метранидозол

д) тетрациклин

013. Активен против атипичных возбудителей

(микоплазма, хламидии, легионелла):

а) гентамицин

б) эритромицин

в) ампиокс

г) левомицетин

д) клиндамицин

014. Котримоксазол является препаратом выбора при лечении:

а) пневмоцистной пневмонии у больных с иммунодефицитом

б) дифтерии

в) холангита

г) пневмококковой пневмонии

д) амебной дизентерии

015. Какой антибиотик противопоказан больным,

получающим миорелаксанты или при миастении?

а) ампициллин

б) гентамицин

в) эритромицин

г) линкомицин

д) ципрофлоксацин

016. Больному бронхиальной астмой,

получающему постоянно преднизолон внутрь, теопек, ингаляции беротека,

в связи с присоединившейся бронхолегочной инфекцией

были назначены эритромицин и бромгексин.

На третий день лечения у больного появилась головная боль, беспокойство,

раздражительность, сердцебиение, ощущение перебоев в сердце,

снижение АД, повышение температуры, тошнота, рвота.

С токсическим действием какого препарата связаны эти симптомы?

а) преднизолон

б) теопек

в) эритромицин

г) беротек

д) бромгексин

017. Концентрацию теофиллина в крови снижают все указанные препараты,

а) фенобарбитала

б) рифампицина

в) карбамазипина

г) нифедипина

д) фенитоина

018. Средняя терапевтическая концентрация теофиллина в плазме составляет:

а) 10-20 мкг/мл

б) 25-30 мкг/мл

в) 30-35 мкг/мл

г) 35-40 мкг/мл

д) 5-10 мкг/мл

019. При сочетанном действии теофиллина и циметидина действие эуфиллина:

а) усиливается

б) усиливается или не изменяется

в) уменьшается

г) уменьшается или не изменяется

д) не изменяется

020. Длительность действия ксантинов:

а) 1-2 часа

б) 2-3 часа

в) 3-4 часа

г) 6-8 часов

д) 10-12 часов

021. Какой механизм действия агонистов 2-адренорецепторов?

а) ингибирование фосфодиэстеразы

б) торможение дегрануляции тучных клеток

в) блокада гистаминовых рецепторов

г) угнетение действия лейкотриенов на дыхательные пути

д) активация аденилатциклазы, увеличение образования цАМФ

022. Укажите начало, максимум действия и длительность действия

фенотерола (беротека):

а) немедленно, 10 минут, 6 часов

б) 15 минут, 30 минут, 6 часов

в) 2-3 минут, 20 минут, 2,5 часа

г) 5-10 минут, 30 минут, 6 часов

д) 30-40 секунд, 20 минут, 3-5 часов

023. Укажите наиболее частые побочные эффекты

ингаляторных форм глюкокортикоидов:

а) развитие остеопороза

б) гиперкортицизм

в) кандидоз полости рта и глотки

г) артериальная гипертония

024. В отличие от беклометазона пропионата, будезонид обладает:

а) более значительным сродством к рецепторам в легких,

подвергается активной биотрансформации в печени

при первом прохождении

б) в большей степени угнетает образование гидрокортизона

в) более часто ведет к развитию гипергликемии

г) более часто вызывает обострение бронхо-легочной инфекции

д) различие между препаратами отсутствует

025. Укажите 2-адреномиметик длительного действия:

а) сальбутамол

б) тербуталин

в) фенотерол

г) орципреналина сульфат

д) формотерол

026. Укажите препарат, обладающий наиболее мощным угнетающим действием

на желудочную секрецию:

а) омепразол

б) циметидин

в) фамотидин

г) сукральфат

для лечения рецидива язвенной болезни 12-перстной кишки:

028. Больной, страдающий ревматоидным полиартритом,

длительное время получает НПВП.

Какой препарат показан этому пациенту

для предотвращения язвообразования?

а) сукральфат

б) гастроцепин

в) ранитидин

г) маалокс

д) мисопростол

029. При лечении хеликобактериоза наиболее эффективно применение:

а) ранитидина

б) оксациллина

в) де-нола

г) де-нола+ампициллин (амоксициллин)

д) маалокса

030. Наиболее рациональный режим назначения антацидов

у больных язвенной болезнью:

а) за 20 минут до еды

б) сразу после еды

в) через 20 минут после еды и на ночь

г) через час после еды и на ночь

д) независимо от приема пищи 4-5 раз в день

031. Какие гипотензивные препараты

действуют преимущественно как постганглионарные адреноблокаторы?

а) пентамин

б) клонидин

в) гуанетидина сульфат

г) анаприлин

д) хлорталидон

032. Какие гипотензивные препараты

действуют на нейрогуморальные механизмы регуляции АД?

а) клонидин

б) каптоприл

в) миноксидил

г) гуанетидин

д) гидрохлортиазид

033. Укажите механизмы гипотензивного действия верошпирона:

а) снижение активности ренина плазмы

б) блокада адренорецепторов

в) уменьшение объема циркулирующей жидкости

г) снижение общего периферического сопротивления

д) конкурентный антагонист альдостерона

034. Биодоступность нифедипина низкая из-за:

а) пресистемной элиминации в печени

б) низкой абсорбции

в) связывания с белками плазмы

г) инактивации в желудочно-кишечный тракт

035. Укажите длительность гипотензивного действия разовой дозы клофелина

при приеме внутрь:

а) 1-2 часа

б) 6-8 часов

в) 10-12 часов

г) 2-24 часа

д) до 3 суток

036. Купирование неосложненного гипертонического криза следует начинать:

а) с внутримышечного введения 1,0 мл 0,01% раствора клофелина

б) с 40 мг фуросемида внутрь

в) с 10-20 мг нифедипина сублингвально

г) с 40 мг анаприлина внутрь

д) с внутривенного 1,0 мл 0,5% раствора фентоламина

037. Каптоприл противопоказан больным:

а) с циррозом печени

б) с хронической почечной недостаточностью

в) с легочным сердцем

г) с сахарным диабетом

д) с язвенной болезнью

038. Укажите механизм действия нитроглицерина:

а) блокада -адренорецепторов

б) спазмолитическое, миотропное действие

на гладкие мышцы сосудистой стенки

в) блокада мелких кальциевых каналов клеточной мембраны

г) повышение активности -рецепторов

сосудистой стенки коронарных артерий

д) повышает медленный ток кальция в клетку

039. Каковы противопоказания к назначеню нитратов?

а) острый инфаркт миокарда

б) артериальная гипертензия

в) гипотония

г) брадикардия

д) атриовентрикулярная блокада

040. Чем обусловлен антиангинальный эффект -адреноблокаторов?

а) расширение коронарных сосудов

б) снижение послеи преднагрузки на сердце

в) уменьшение работы сердца

г) центральный механизм действия

д) повышение потребности миокарда в кислороде

041. Укажите -адреноблокатор селективного действия:

а) обзидан

б) тразикор

в) вискен

г) сектраль

042. Какова биоусвояемость пропранолола при приеме внутрь?

043. Какова биодоступность верапамила при приеме внутрь?

044. Укажите противопоказания к назначению нифедипина:

а) артериальная гипертония

б) сердечная недостаточность

в) бронхиальная астма

г) артериальная гипотония

д) атриовентрикулярная блокада 2-й степени

045. Укажите препарат, относящийся к 1-й группе антиаритмических средств

(мембраностабилизирующего действия):

а) лидокаин

б) изоптин

в) кордарон

г) хинидин

д) дифинин

046. Укажите препарат, относящийся ко 2-й группе антиаритмических средств

(местные анестетики):

а) мекситил

б) обзидан

в) вискен

г) кордарон

д) новокаинамид

047. Укажите препарат, относящийся к 3-й группе антиаритмических средств

(-адреноблокаторы):

а) лидокаин

б) тразикор

в) кордарон

г) хинидин

д) изоптин

048. Укажите продолжительность действия лидокаина:

а) 20 минут

б) 60 минут

в) 1,5-2 часа

д) 12 часов

049. Укажите период полувыведения кордарона:

а) 4-6 часов

б) 1-2 часа

в) 20-24 часа

050. Укажите время достижения максимальной концентрации изоптина

в плазме крови при приеме внутрь:

а) 10 минут

б) 50 минут

в) 1,5-2 часа

г) 8 часов

д) 10 часов

051. Укажите местный анестетик,

обладающий наиболее выраженным антиаритмическим действием:

а) лидокаин

б) тримекаин

в) ксикаин

г) новокаин

д) мекситил

052. Развитие толерантности к нитратам зависит, главным образом:

а) от пути введения нитрата

б) от времени достижения максимальной концентрации в крови

в) от продолжительности действия

г) от комбинации с другими лекарствами

д) ни один из перечисленных факторов

не влияет на развитие толерантности

053. В отличие от изосорбид-динитрата, 5-изосорбид-мононитрат:

а) не подвергается первичной пресистемной элиминации

при прохождении через печень

б) не вызывает развития толерантности

в) не вызывает метгемоглобинии

г) не вызывает головной боли

д) различия между препаратами отсутствуют

054. Назовите противовоспалительное средство пролонгированного действия:

а) аспирин

б) анальгин

в) пироксикам

г) индометацин

д) ортофен

055. При приеме салицилатов их наименьшая концентрация наблюдается:

а) в почках

б) в печени

в) в миокарде

г) в легких

д) в головном мозге

056. Назовите глюкокортикоидные препараты с пролонгированным действием:

а) преднизолон

б) полькортолон

в) дексаметазон

г) кеналог

д) метилпреднизолон

057. Укажите побочное действие никотиновой кислоты:

а) липодистрофия

б) гиперурикемия

в) рабдомиоз

г) нарушение зрения

д) бронхоспазм

058. Укажите побочное действие секвестрантов желчных кислот:

а) кожный зуд

б) диарея

в) запоры

г) нарушение зрения

д) депрессия

059. Какой антигистаминный препарат противопоказан

а) супрастин

б) пипольфен

в) димедрол

г) тавегил

д) фенкарол

060. Какой антигистаминный препарат противопоказан

при лечении аллергических реакций на введение ЛС,

а) пипольфен

б) супрастин

в) димедрол

г) тавегил

д) фенкарол

061. Период полувыведения лекарств - это:

а) время достижения максимальной концентрации лекарства в плазме

б) время, в течение которого лекарство достигает системного кровотока

в) время, в течение которого лекарство распределяется в организме

г) время, за которое концентрация лекарства в плазме снижается на 50%

д) время, за которое половина введенной дозы достигает органа-мишени

062. Терапевтический индекс - это:

а) терапевтическая доза лекарства

б) отношение концентрации лекарства в органе или ткани

к концентрации его в плазме крови

в) диапазон между минимальной и максимальной

г) процент не связанного с белком лекарства

д) диапазон между минимальной и максимальной

терапевтическими концентрациями лекарства

063. К рецепторным средствам конкурентного действия относятся:

а) нестероидные противовоспалительные средства

б) -адреноблокаторы

в) петлевые диуретики

г) нитраты

д) фторхинолоны

064. При назначении следующих лекарственных средств

следует учитывать функцию и печени и почек:

а) липофильных, образующих неактивные метаболиты

б) липофильных, образующих активные метаболиты

в) гидрофильных

г) гепатотоксичных

д) нефротоксичных

065. Селективность действия лекарственного вещества зависит:

а) от периода полувыведения

б) от способа приема

в) от связи с белком

г) от объема распределения

д) от дозы

066. Строго дозозависимыми являются следующие группы побочных эффектов:

а) фармацевтические

б) токсические

в) аллергические

г) мутагенные

д) синдром отмены

067. Перечислите группы препаратов с узким терапевтическим индексом:

а) -блокаторы

б) пенициллины

в) сердечные гликозиды

г) метилксантины

д) мощные диуретики

068. Препаратом выбора при наличии атипичных возбудителей

(микоплазма, хламидии) является:

а) эритромицин

б) метронидазол

в) гентамицин

г) карбенициллин

д) цефуроксим

069. Препаратами выбора при наличии атипичных возбудителей

(микоплазма, хламидии) являются:

а) макролиды

б) пенициллины

в) аминогликозиды

г) цефалоспорины

д) сульфаниламиды

070. Укажите антибактериальный препарат,

обладающий наибольшей антианаэробной активностью:

а) эритромицин

б) ампициллин

в) тетрациклин

г) гентамицин

д) цефотетан

071. Дисбактериоз кишечника

вызывают все перечисленные антибактериальные препараты, кроме:

а) полусинтетических препаратов

б) тетрациклинов

в) фторхинолонов

г) оральных цефалоспоринов

д) макролиды

072. Нефротоксичными являются следующие антибактериальные препараты,

а) гентамицина

б) карбенициллина

в) эритромицина

г) цефазолина

д) ванкомицина

073. Укажите антибактериальный препарат,

не активный в отношении пневмококка:

а) азитромицин

б) пенициллин

в) цефтриаксон

г) ципрофлоксацин

д) левомицетин

074. Выберите комбинацию антибактериальных препаратов,

обладающую синергизмом действия и безопасностью:

а) пенициллины + тетрациклины

б) пенициллины + цефалоспорины

в) пенициллины + макролиды

г) пенициллины + аминогликозиды

д) пенициллины + сульфаниламиды

075. Хорошо проникают через гематоэнцефалический барьер

следующие антибактериальные препараты:

а) пенициллины

б) макролиды

в) тетрациклины

г) аминогликозиды

д) цефалоспорины

076. Препаратом выбора при крупозной пневмонии является:

а) цефаклор

б) доксициклин

в) метициллин

г) цефотаксим

д) пенициллин

077. Препаратом выбора при фарингите является:

а) цефаклор

б) тетрациклин

в) цефтазидим

г) офлоксацин

д) пенициллин

078. Новое поколение макролидных антибиотиков

имеет следующее преимущества:

а) высокая биодоступность

б) широкий спектр антибактериального действия

в) бактерицидное действие

г) длительный период полувыведения

д) почечный путь экскреции

079. Фторхинолоны отличаются от хинолонов следующими свойствами:

а) широким антибактериальным спектром действия

б) бактериостатическим действием

в) высокой пенетрацией в ткани

г) постантибактериальным эффектом

д) пероральным способом введения

080. Выберите препарат,

максимально подавляющий секрецию соляной кислоты:

а) пирензепин

б) циметидин

в) карбеноксолон

г) антациды

д) омепразол

081. Максимальное количество побочных эффектов среди Н2-блокаторов

вызывает:

а) циметидин

б) роксатидин

в) низатидин

г) ранитидин

д) фамотидин

082. Синтетические аналоги простагландинов (энпростил, мизопростол)

вызывают перечисленные эффекты:

а) антисекреторное действие

б) секрецию барбитуратов

в) слизеобразование

г) репарантное действие

083. Угнетает метаболизм других препаратов:

а) омепразол

б) карбеноксолон

в) циметидин

г) фамотидин

д) гастроцепин

084. Продолжительность антисекреторного действия омепразола составляет:

а) 2-4 часа

б) 8-10 часов

в) 16-20 часов

д) 3 суток

085. Укажите препарат,

являющийся мощным стимулятором слизеобразования в желудке:

а) карбеноксолон

б) платифилин

г) омепразол

д) метоклопрамид

086. При наличии почечной недостаточности

требуется коррекция доз препаратов:

а) аналогов простогландинов

б) омепразола

в) Н2-блокаторов

г) сукральфата

д) холиноблокаторов

087. Укажите антисекреторный препарат, блокирующий "протоновый насос"

а) метоклопрамид

б) карбеноксолон

в) пирензепин

г) сукральфат

д) омепразол

088. При патологии почек

возникают следующие изменения фармакогенетики лекарств:

а) нарушение почечной экскреции

б) увеличение концентрации лекарств в плазме крови

в) уменьшение связывания с белками плазмы

г) увеличение периода полувыведения

д) уменьшение биодоступности

089. Цирроз печени вызывает следующие изменения фармакокинетики лекарств:

а) снижение пресистемного метаболизма

б) уменьшение связывания с белками плазмы

в) увеличение периода полувыведения

г) увеличение биодоступности

д) уменьшение объема распределения

090. При сердечной недостаточности

наблюдаются следующие изменения фармакокинетики дигоксина:

а) снижение абсорбции в желудочно-кишечном тракте на 30%

б) уменьшение связывания с белком плазмы

в) усиление метаболизма в печени

г) снижение почечной экскреции

д) увеличение периода полувыведения

091. Алкоголь приводит:

а) к увеличению абсорбции лекарств

в) к замедлению метаболизма в печени

г) к снижение почечной экскреции

д) к увеличение периода полувыведения

092. Никотин приводит:

а) к уменьшению абсорбции лекарств

б) к увеличению объема распределения лекарств

в) к увеличению связи с белками плазмы

г) к ускорению метаболизма в печени

д) к усилению почечной экскреции лекарств

093. При стенокардии напряжения показан:

а) нифедипин

б) пропранолол

в) капотен

г) эналаприл

д) клонидин

094. При стенокардии Принцметала (вазоспастической) показан:

а) нифедипин

б) обзидан

в) дипиридамол

г) допегит

д) каптоприл

095. Критерием эффективности антиангинального средства является:

а) прирост времени нагрузки на ВЭМ >1 минуты

б) увеличение количества потребляемого НТГ

в) прирост времени ВЭМ - пробы >2 минут

г) снижение времени нагрузки

д) переход больного из 2-го в 3-й функциональный класс стенокардии

096. К антиангинальным средствам относятся:

а) курантил

б) капотен

в) аспирин

г) верапамил

097. Для купирования приступа стенокардии применяют:

а) сустак

б) нитронг

в) нитроглицерин

г) верапамил

д) дилтиазем

098. Из антиангинальных средств

при сочетании ИБС и артериальной гипертонии показан:

а) сустак

б) верапамил

в) каптоприлил

г) курантил

д) эналаприл

099. Наиболее вероятно развитие толерантности при применении:

а) тринитролонга

б) сустака

в) сублингвального нитроглицерина

г) изосорбита-5-мононитрата

д) нитронга

100. Методом контроля эффективности антиангинальной терапии является:

а) мониторирование ЭКГ по Холтеру

б) контроль уровня липидов крови

в) суточное мониторирование АД

г) измерение ФВД (функции внешнего дыхания)

д) измерение АД в орто- и клиностазе

101. Препаратом выбора при стенокардии у больного с брадикардией является:

а) пиндолол

б) пропранолол

в) верапамил

г) дилтиазем

д) метопролол

102. Препаратом выбора при стенокардии

у больного с сердечной недостаточностью является:

а) верапамил

б) коринфар

в) дилтиазем

г) ацебуталол

д) нитросорбит

103. К ингаляционным кортикостероидам относятся:

а) гидрокортизон

б) беклометазон

в) преднизолон

г) полькортолон

д) дексаметазон

104. К селективным 2-агонистам длительного действия относится:

а) флутиказон

б) сальметерол

в) сальбутамол

г) фенатерол

д) тербуталин

105. Для купирования приступа бронхиальной астмы применяется:

а) ипратропия бромид

б) теопек

г) сальбутамол

д) задитен

106. Мембраностабилизатором для приема внутрь является:

а) кетотифен

б) недокрамил натрия

в) хромогликат натрия

г) супрастин

д) ипратропиум бромид

107. При синдроме "запирания" применяют:

а) сальбутамол

б) фенотерол

в) теопек

г) эуфиллин

д) адреналин

108. К муколитическим средствам относится:

а) кодеин

б) хромогликат натрия

в) ацетилцистеин

г) сальметерол

д) теофиллин

109. При одновременном применении

повышает концентрацию теофиллина в крови:

а) офлоксацин

б) пенициллины

в) цефтриаксон

г) гентамицин

д) бисептол

110. При одновременном применении

снижает концентрацию теофиллина в крови:

а) пефлоксацин

б) циметидин

в) рифампицин

г) эритромицин

д) ампиокс

111. При повышеном давлении в легочной артерии

у больного с бронхиальной астмой показан:

а) верапамил

б) нифедипин

в) дигоксин

д) беклометазон

112. При бронхиальной астме на фоне хронического бронхита показан:

а) ипратропия бромид

б) адреналин

в) эфедрин

г) кетотифен

д) супрастин

113. Укажите предпочтительный путь введения лекарственных препаратов

при застойной сердечной недостаточности:

а) ректальный

б) сублингвальный

в) внутрь

г) внутривенный

д) накожный

114. Перечислите препараты,

обладающие прямым положительным инотропным эффектом:

а) дигоксин

б) допамин

в) норадреналин

г) эуфиллин

д) гидралазин

115. Укажите состояния,

повышающие чувствительность к сердечным гликозидам:

а) пожилой возраст

б) тиреотоксикоз

в) легочное сердце

г) гипокалемия

д) застойная сердечная недостаточность

116. Перечислите препараты, при взаимодействии с которыми

концентрация дигоксина в крови может повышаться:

а) фосфулагель

б) хинидин

в) верапамил

г) амиодарон

117. Перечислите факторы, замедляющие всасывание сердечных гликозидов

из желудочно-кишечного тракта:

а) хроническая почечная недостаточность

б) застойная сердечная недостаточность

в) язва желудка

г) совместный прием с антацидными средствами

118. Перечислите факторы,

обеспечивающие наибольшую безопасность и эффективность диуретиков

при длительном лечении сердечной недостаточности:

а) максимальные дозы

б) средние дозы

в) минимальные дозы

г) ежедневный прием

д) прерывистый прием

119. Укажите наиболее эффективный диуретик

для лечения застойной сердечной недостаточности

с развитием вторичного гиперальдостеронизма:

а) этакриновая кислота (урегид)

б) хлорталидон (гигротон)

в) ацетазоламид (диакарб)

г) спиронолактон (верошпирон)

д) триампур

120. Укажите основной терапевтический эффект нитросорбида

у больных с сердечной недостаточностью:

а) расширение преимущественно артериол и снижение постнагрузки

б) расширение преимущественно венул и снижение преднагрузки

в) прямое положительное инотропное действие

г) повышение диуреза и снижение преднагрузки

121. Перечислите препараты, имеющие холинолитические побочные эффекты:

а) лидокаин

б) хинидин

в) амиодарон (кордарон)

г) верапамил

д) прокаинамид (новокаинамид)

122. Перечислите группы антиаритмических средств,

оказывающих антифибрилляторное действие:

а) сердечные гликозиды

б) антагонисты кальция (4-я группа)

в) -блокаторы (2-я группа)

г) амиодарон, бретилия тозилат (3-я группа)

д) хинидин, прокаинамид и другие препараты 1а группы

123. Перечислите препараты,

которые могут спровоцировать приступ предсердной тахиаритмии

при синдроме Вольфа-Паркинсона-Уайта:

а) дигоксин

б) амиодарон

в) верапамил

г) пропранолол

д) этмозин

124. Укажите препараты, которым свойственен аритмогенный эффект:

а) аймалин

б) мексилетин

в) пропафенон

г) амиодарон

125. Перечислите показания к лечению антиаритмическими препаратами:

а) аритмии, вызывающие нарушение гемодинамики

б) субъективная непереносимость аритмии

в) нарушение ритма высоких градаций

г) частые нарушения ритма

126. Перечислите препараты,

основное антиаритмическое действие которых

связано с удлинением атриовентрикулярной проводимости:

а) пропранолол

б) лидокаин

в) верапамил

г) дигоксин

д) прокаинамид

127. Укажите эффекты взаимодействия амиодарона и дизопирамидов

(ритмилен, норпейс):

а) замедление метаболизма дизопирамида

б) замедление метаболизма амиодарона

в) повышение риска развития побочных эффектов дизопирамида

г) повышение риска развития побочных эффектов амиодарона

128. Укажите побочный эффект нифидепина:

а) брадикардия

б) бронхоспазм

в) отеки голеней и стоп

г) развитие F-V блокады

д) ульцерогенность

129. Укажите препарат,

механизмом гипотензивного действия которого

является блокада -рецепторов:

а) клофелин

б) празозин

в) пропранолол

г) капотен

д) верапамил

130. Препаратом выбора

у больного с гипертонией и застойной недостаточностью кровообращения

является:

а) эналаприл

б) нитросорбид

в) клофелин

г) адельфан

д) пентамин

131. Оптимальный препарат для длительной гипотензивной терапии должен:

а) влиять на обмен веществ

б) иметь рикошетные реакции

в) обладать синдромом отмены

г) иметь стабильную концентрацию в крови

д) вызывать ортостатические реакции

132. Перечислите группы гипотензивных препаратов,

уменьшающих активность ренин-ангиотензин-альдостероновой системы:

а) ингибиторы АПФ

б) -блокаторы

в) центральные -агонисты

г) тиазидные диуретики

д) антагонисты кальция

133. Укажите гипотензивные препараты,

которые необходимо с осторожностью применять

при сочетании сахарного диабета и гипертонической болезни:

а) верапамил

б) пропранолол

в) дилтиазем

г) гипотиазид

д) эналаприл

134. Проведение лекарственного мониторинга

требуется при лечении следующими группами препаратов:

а) противосудорожными

б) 2-симпатомиметиками

в) метилксантинами

г) глюкокортикоидами

д) М-холиномиметиками

135. Развитие асистолии возможно при комбинации пропранолола:

а) с фенобарбиталом

б) с фуросемидом

в) с верапамилом

г) с фенитеином

д) с циметидином

136. Риск токсических эффектов увеличивается при комбинации гентамицина:

а) с фуросемидом

б) с пенициллином

в) с метилксантинами

г) с макролидами

д) с глюкокортикоидами

КЛИНИЧЕСКАЯ ФАРМАКОЛОГИЯ

114 - а,б,в,г,д

115 - а,б,в,г,д

088 - а,б,в,г

089 - а,б,в,г

090 - а,б,г,д

124 - а,б,в,г,д

078 - а,б,в,г

079 - а,в,г,д

083 - а,б,в,г

Выберите правильное утверждение: а)биодоступность-количество ЛС,поступающее в системный кровоток,выраженное в процентах от введенной дозы,б)биодоступность определяется величиной адсорбции ЛС в ЖКТ и выраженностью эффекта первого прохождения через печень.в)биодоступность определяют по формуле: F = AUC (в/м или внутрь)/AUC (в/в).г)биодоступность ЛС при внутримышечном введении определяется степенью его всасывания и биотрансформации в организме.
Ответ: а, б, в

2.
Ответ: Атровент

3.

Ответ: а, д

4.

Ответ:

5.
Ответ:

6.

д) Ксилит
Ответ: а,в

7.

Ответ: а,б,д

8.
Ответ:

9. поступил с острым инфарктом миокарда,возникшим 5ч назад.Назначения: анаприлин 20 мг 4 раза в сутки внутрь, гепарин в/в капельно по 10 000 ЕД каждые 4 часа.При этом удалось достигнуть увеличения времени свертывания крови до 18-23 минуты. На следующий день у б-ного диагностирована правосторонняя нижнедолевая пневмония.Назначена натриевая соль бензилпенициллина (по 1000 000 ЕД каждые 4 ч) в/в.Через 4 ч время свертывания крови составило 8 минут. Какова ваша тактика?
Ответ:

10.

11.
Ответ: Вит.В12 в дозе 500 мкг/сут через день, фолиевая кислота в дозе 1,5 мг/сут, сульфат железа(80 мг Fe2+) 1 раз в сутки

12.

Ответ: Вит.С

13.

Ответ: Церебролизин

14.
аллергию (на бутадион, гепарин, метиндол, пенициллин, теофиллин).В стаци-онаре б-ной назначены реопирин по 5 мл в/м 1 раз в сутки,гидрокортизона гемисукцинат по 100 мг в полость коленных суставов,тавегил по 0,001 г 2р.в сутки.Через 3 дня у б-ной появились зудящие эритематозные высыпания на коже туловища.Какова наиболее вероятная причина
ухудшения состояния?
Ответ:

15.



Ответ: а,б,д,е,з,и

16.
Ответ: Через несколько месяцев

17.


Ответ: а,б,в,д,е

18.
Ответ: а,б,в,г,д,ж,з

19.
Ответ:

20.
Ответ: Ципрофлоксацин

21. Феномен первого прохождения ЛС через печень зависит от:а) кровоснабжения печени, б) связывания ЛС с белком, в) активности ферментов гепатоцитов, г) уровня экскреции ЛС,д) быстроты всасывания
Ответ: а, в

22. Препараты, влияющие на микросомальные ферменты печени:индукторы микросомальных ферментов печени: а) пенициллин, б) нитроглицерин, в) фенобарбитал, г) фуросемид, д) бутадион, е) кортизол, ж) пропранолол, з) циметидин, и) левомицетин, к) дифенин
Ответ: в,д

23. поступила в отделение с ж-ми на боли в правой молочной железе, повышение Т.до 39,5 С.Заболела 3 дня назад,на 10-й день после родов. При поступлении в отделение в верхненаружном квадранте правой молочной железы обнаружена гиперемия кожи,массивный инфильтрат с флуктуацией в центре.Диагноз:острый правосторонний мастит.Б-ная оперирована. Взят посев отделяемого раны.Определите антибиотик первого выбора
Ответ: Цефазолин

24.

Ответ: Анафилактическая реакция

25.

Ответ: Левомицетин

26.
Ответ:

27.
Ответ: Бигуаниды

28.

Ответ: Гипотония, головокружение.

29.

Оцените действия врача.

30. У больного Д., 53 лет, диагноз: ИБС, стабильная стенокардия??? ФК, постинфарктный кардиосклероз, мерцательная аритмия,ХНК??Б ст. Принимал строфантин, дигоксин, фуросемид, панангин в средних терапевтических дозах. Неожиданно у больного повысилась температура до 38,4°С, появилась кашель, одышка, крепитация в легких справа. На рентгенограмме легких справа в нижней доле определяется участок инфильтрации. К лечению добавлены гентамицин, сульфокамфокаин,супрастин.

Ответ:

31.

Ответ: Фентоламин.

32.

Ответ: г,д

33.

Ответ:

34.
Ответ:

35.
Ответ: Эналаприл.

36.

Ответ: а,б,г

37.

Ответ:

38.
Ответ:

39.
Ответ:

40.

Ответ: все перечисленное

41. Препараты, влияющие на микросомальные ферменты печени: ингибиторы микросомальных ферментов печени:
а) пенициллин, б) нитроглицерин,в) фенобарбитал,
г) фуросемид, д) бутадион, е) кортизол, ж)пропранолол,
з) циметидин, и) левомицетин, к) дифенин
Ответ: з,и

42.
Ответ: Через 7-14 дней

43. Укажите комбинацию препаратов, приводящую к возникновению конкуренции за связывание с белком, что может обусловить опасное повышение содержания свободной фракции одного из препаратов в крови и появление симптомов его передозировки:
Ответ: неодикумарин и бутадион

44. Выберите ЛС с узким терапевтическим диапазоном:
а) пенициллины, б) противосудорожные средства,
в) антиаритмические средства,г) дигоксин, д) метотрексат, е) теофиллин, ж) циклоспорин, з) макролиды
Ответ: б,в,г,д,е,ж

45. Укажите комбинации препаратов, при которых вследствие конкуренции за связь с белком происходит увеличение концентрации в плазме крови свободной фракции одного из них: а.строфантин и мисклерон, б.дигитоксин и мисклерон, в.неодикумарин и бутадион, г.нифедипин и гидрохлортиазид
Ответ: б,в

46. выявлены частая желудочковая экстрасистолия и пароксизмы мерцательной аритмии.ЧСС 74 в мин,АД 140/80 мм.рт.ст.Последние 3г.беспокоят приступы стенокардии напряжения и покоя.Лечение проводили кордароном.Учитывая побочные эффекты назначенного препарата,выберите ЛС для дальнейшего лечения б-ного: а) Хинидин,
б) Боннекор, в) Этацизин,

Ответ: а,б

47. Известно,что при комбинации хинидина и дигоксина часто наблюдается гликозидная интоксикация.С чем она связана? Фармакодинамическое взаимодействие:
Ответ: синергизм

48. Известно,что при комбинации хинидина и дигоксина наблюдается гликозидная интоксикация.С чем она связана? Фармакокинетическое взаимодействие, влияние хинидина на:
Ответ: связь с белком

49. Критические периоды внутриутробного развития:
а. период предимплантационного развития (1 нед)
б. стадия эмбриогенеза заканчивается к 8 нед.
в. стадия эмбриогенеза заканчивается к 8 мес.
г. период непосредственно перед родами
Ответ: а,б,г

50. Выберите из нижеуказанных препаратов,обладающие перечисленными свойствами: Противомикробные препараты,применение которых практически безопасно при беременности: а.сульфаниламды,в том числе бисептол,
б.аминогликозиды,тетрациклины,рифампицины,метронидазол (в 1триместр беременности), в. пенициллины, цефалоспорины, эритромицин,линкомицин,фузидин, г.антимикотические средства,противоопухолевые
антибиотики.
Ответ: в

51. Метронидазол назначен кормящей матери, укажите побочные эффекты:
а.повышение возбудимости,тахикардия, б.угнетение аппетита,рвота, в.угнетение ЦНС,дыхания,снижение массы тела, г.увеличение секреции пролактина,нагрубание молочных желез, д.гипоплазия надпочечников, нарушение обмена веществ,повышение риска развития билирубиновой энцефалопатии, е.геморрагии,нарушение дыхания,ацидоз, ж.угнетение кроветворения,анемия, гипотрофия, дисбактериоз.
Ответ: б

52. Антимикробные препараты первого выбора у новорожденных: а.бензилпенициллин,оксациллин,карбенициллин,гентамицин,амикацин,б.бензилпенициллин,оксациллин,бициллины,цефазолин,цефотаксим,эритромицин,линкомицин,нистатин, в.карбенициллин,гентамицин,сизомицин,амикацин, тобрамицин,цепорин (при неэффективности цефалоспоринов первого поколения), г.эритромицин,линкомицин,нистатин,леворин,карбенициллин,
гентамицин, сизомицин
Ответ: б

53.
Ответ:

54. Основные особенности фармакокинетики ЛС у лиц пожилого возраста:
а.снижение скорости абсорбции, б.ускорение абсорбции, в.снижение скорости распределения, г.ускорение распределения, д.уменьшение связывания ЛС с белками плазмы, е.увеличение связывания ЛС с белками плазмы, ж.замедление метаболизма, з.ускорение метаболизма,
и.замедление выведения ЛС, к. ускорение выведения ЛС.
Ответ: а,в,д,ж,и

55.
Ответ: б,в,г

56. Укажите побочные эффекты бета-адреноблокаторов: а)брадикардия, б)артериальная гипотензия, в)бронхоспазм, г)тахикардия, д)нарушение функций щитовидной железы,
е) перемежающаяся хромота, ж) АВ-блокада
Ответ: а,б,в,е,ж

57.
состояния: а)Естественное течение заболевания, б)Развитие толерантности к нитратам, в)Синдром межкоронарного обкрадывания, г) Возникновение синдрома рикошета д) явления идиосинкразии
Ответ: а,б

58. Укажите побочные эффекты амиодарона: а) брадикардия, б).артериальная гипотензия, в) бронхоспазм,г)тахикардия, д) нарушение функций щитовидной железы, е) перемежающаяся хромота, ж) АВ-блокада
Ответ: а,в,д,ж

59. Как изменится ваша антиангинальная терапия,если у б-ного на фоне терапии нитратами возникнет инсульт головного мозга?
Ответ: отмена нитратов и назначение антиангинального препарата другой группы

60. Какие гипотензивные препараты считают наиболее безопасными для пожилых больных: а)бета-адреноблокаторы, б)ганглиоблокаторы, в)симпатолитики, г)блокаторы медленных кальциевых каналов, д)тиазидные
диуретики, е) ингибиторы АПФ.
Ответ: г,д

61. Схема лечения кордароном:
Ответ: по схеме, предполагающей постепенное снижение дозы с 600 мг до 200 мг в сутки

62. Как влияют ингибиторы МАО (антидепрессанты) на прессорный эффект адреностимуляторов прямого и непрямого действия?
Ответ: усиливают действие

63. Небензодиазепиновый» агонист бензодиазепиновых рецепторов:
Ответ: Золпидем

64. Снотворное средство — соединение алифатического ряда:
Ответ: Хлоралгидрат

65.

Ответ: А (б)

66. Протамина сульфат назначают при передозировке:
Ответ: Гепарина

67. Какой способ детоксикации наиболее эффективен при отравлении веществами, связывающими с белками и липидами крови?
Ответ: Гемосорбция

68. Принцип действия налоксона при остром отравлении морфином:
Ответ: Препятствует действию морфина на опиоидные рецепторы

69. Укажите препараты,обладающие свойствами антиоксидантов: а)верапамил б) вит.А, в) вит.К, г) вит.С, д) вит.Е, е)селен, ж) карнозин, з) доксициклин
Ответ: б,г,д,е,ж

70. Какие эффекты характерны для нейролептиков?
а) антипсихотический, б) седативный, в) противорвотный
Ответ: а,б,в

71. У Б-ой 64л.развился острый приступ закрытоугольной глаукомы с сильными болями в правом глазу с иррадиацией в голову. Появились тошнота и рвота,одышка,выявлены признаки гипертонического криза 2 типа с ЧСС 62
в мин.АД 200/140 мм.рт.ст.Б-ная в течение многих лет страдает гипертонической болезнью.В легких большое кол-во влажных мелкопузырчатых хрипов.Какие диуретики показаны больной? а. Клопамид, б.Верошпирон,в.Гипотиазид, г.Фуросемид в/в, д. Диакарб:
Ответ: г,д

72. в течение 15 лет страдает сахарным диабетом,по поводу которого получает инсулин по 70 ЕД/сут,что поддерживает уровень гликемии в пределах 7,5-8,6ммоль/л.В последнее время стало повышаться АД до
170/90-180/100 мм.рт.ст.в связи с чем лечащий врач назначил обзидан в суточной дозе 120 мг.Какие побочные эффекты следует ожидать при данной комбинации ЛС? а.Гипергликемия вплоть до комы, б.Сердечная недостаточность, в.Гипогликемия вплоть до комы, г.Ортостатическая гипотензия, д. Гипертензия
Ответ: б,в

73. , страдающему гормонально-зависимой бронхиальной астмой,назначили преднизолон (по 5 мг ежедневно),сальбутамол (ингаляция 2 доз аэрозоля 4р.в сутки).В связи с проявлениями судорожного синдрома (в анамнезе черепно-мозговая травма) был назначен фенобарбитал.Через неделю у б-го развилось обострение бронхиальной астмы.С чем это связано?
А. Фенобарбитал ускорил биотрансформацию: а.сальбутамола, б.преднизолона, Б.Фенобарбитал ускорил экскрецию: а.сальбутамола, б.преднизолона, В.Фенобарбитал замедлил экскрецию: а.сальбутамола, б.преднизолона, Г.Фенобарбитал замедлил биотрансформацию:а.сальбутамола,б.преднизолона
Ответ: А (б)

74. страдает ИБС,стенокардией напряжения III ФК.ЧСС 90 в мин,АД 150/80мм.рт.ст.В анамнезе-хронический бронхит с бронхоспастическим синдромом в стадии ремиссии.Жировая дистрофия печени.Укажите группы ЛС (второй этап выбора лекарственной терапии),оптимальные для антиангинальной терапии. а.Нитраты и верапамил, б. Нитраты и атенолол
в) Нитраты и анаприлин, г) Нитраты и нифедипин,
д) Нифедипин и амиодарон
Ответ: а

75. по поводу стенокардии принимает нитросорбид по 10мг 4р.в сутки ЧСС 80 в ми.АД 140/80мм.рт.ст.Через 1 мес.после начала терапии вновь участились приступы стенокардии.Каковы возможные причины ухудшения
состояния: а)Естественное течение заболевания, б)Развитие толерантности к нитратам, в)Синдром межкоронарного обкрадывания, г) Возникновение синдрома рикошета, д) явления идиосинкразии
Ответ: а,б

76. отмечаются приступы стенокардии при умеренных физических нагрузках.В анамнезе коллаптоидное состояние после однократного приема нитроглицерина сублингвально (с тех пор б-ной нитроглицерин не принимал). Сопутствующие заболевания-ГБ (рабочий уровень АД 160/100мм.рт.
ст.,гипофункция щитовидной железы.На момент осмотра АД 190/100мм.рт.ст.,ЧСС 72 в минуту.Больному противопоказаны:
Ответ: Амиодарон

77. по поводу артериальной гипертензии 2степени получает 0,000075г клофелина 4р.в сутки.В связи с развитием сенильной депрессии назначен мелипрамин.Через 3 дня после назначения мелипрамина у б-ного возник гипертонический криз.Накануне б-ной не принимал клофелин.Каковы возможные причины ухудшения состояния: а)Следствие естественного течения заболевания, б)Следствие гипертензивного эффекта мелипрамина, в)Следствие неблагоприятного взаимодействия ЛС, г)Следствие возможного прекращения приема ЛС б-ным и развития синдрома отмены.
Ответ: б,в,г

78. по поводу гипертонического криза вводили натрия нитропруссид в/в капельно в больших дозах (со скоростью 8 мкг/мин). Появились одышка, акроцианоз,давящие боли за грудиной,мышечные подергивания.Какова причина ухудшения состояния б-ного?
Ответ: Токсическое действие цианидов

79. выявлены частая желудочковая экстрасистолия и пароксизмы мерцательной аритмии.ЧСС 74 в мин,АД 140/80 мм.рт.ст.Последние 3г.беспокоят приступы стенокардии напряжения и покоя.Лечение проводили кордароном.Учитывая побочные эффекты назначенного препарата,выберите ЛС
для дальнейшего лечения б-ного: а) Хинидин, б) Боннекор, в) Этацизин,
г) Мекситил, д) Верапамил, е) Пропранолол
Ответ: а,б

80. пароксизмальная суправентрикулярная тахикардия на фоне синдрома WPW.Для купирования приступа выбран аймалин.Определите оптимальную схему лечения выбранным препаратом: а)1 мг/кг в/в в течение более10 мин,при необходимости повторить через 30 мин, б)50 мг в/в в тече-
ние 3-5 мин в 10 мл 5% раствора глюкозы или изотонического раствора NaCl или в/м, в) 0,5-1 г в/в каждые 2 мин вводят по 0,1-0,2 г или в/м
г) после парентерального введения назначить внутрь по 100мг 4-5 раз в сутки, поддерживающая доза 50 мг 3-4 раза в сутки
Ответ: а,г

81. У б-ной 28л.с диагнозом СКВ на фоне ХПН появились отеки голеней,увеличение печени.При эхокардиографическом исследовании определяется снижение сердечного выброса. ЧСС 95 в мин,АД 170/100 мм.рт.ст.Какие сердечные гликозиды показаны больной?
Ответ: Дигитоксин

82. У б-ной 28л.с диагнозом СКВ на фоне ХПН появились отеки голеней,увеличение печени.При эхокардиографическом исследовании определяется снижение сердечного выброса.ЧСС 95 в мин, АД 170/100 мм.рт.ст. Больная принимает дигитоксин.В связи с появлением судорожного синдрома дополнительно назначен фенобарбитал (0,3 г/сут).Когда возникнут изменения в состоянии б-ной при наличии влияния?
Ответ: Через 7-14 дней

83. 57 лет по поводу постинфарктного атерокардиосклероза,застойной сердечной недостаточности 2 степени получает 40 мг фуросемида в/в и 300 мг
верошпирона внутрь.Какую диуретическую терапию Вы назначите б-ному в случае рефрактерности?
Ответ: Фуросемид 80 мг в/в и спиронолактон 300 мг внутрь

84. страдает неатопической бронхиальной астмой,сопровождающейся обильной бронхореей.Пульс 62 в мин.АД 140/80 мм.рт.ст.Какие препараты более предпочтительны?
Ответ: Атровент

85. упорно рецидивирующий синдром бронхиальной обстукции со сниженной чувствительностью к холино и адренотропным средствам.Бронхиальной астмой страдает более 10 лет.Что можно назначить для уменьшения частоты и тяжести приступов бронхиальной астмы: а)Ингаляции бета
2-адреностимуляторов более 6 раз в сутки, б) Ингаляция м-холиноблокатора, в) Введение адреналина п/к в большей дозе,чем обычно,для снятия бронхоспазма, г)Эуфиллин в/в, д) Ингаляционные глюкокортикоиды.
Ответ: г,д

86. поступил с ж-ми на изжогу,боли в эпигастральной области натощак, купируемые приемом натрия гидрокарбоната.При ФЭГДС выявлена язва (0,5см в диаметре) в ампуле 12 п.к.РН-метрия желудочного сока: кислотообра-
зующая функция средней интенсивности с низкими щелочными резервами,холинергический тип рецепции. Диагноз: язвенная болезнь 12 п.к.в стадии обострения. Выберите наиболее эффективное и безопасное ЛС и определите режим его дозирования:
Ответ: Пирензепин до еды по 0,05 г 3 раза в сутки в течение 2дней, затем по 0,05 г 2 раза в сутки

87. выявлена дискенезия желчного пузыря по гипертоническому типу. Выберите оптимальный вариант лечения.
Ответ: Но-шпа по 1-2 табл.3 раза в сутки, отвар бессмертника по 1/2 стакана за 30 мин до еды

88. страдает хрон.холецистопанкреатитом в течение 5 лет.За последнюю неделю после нарушения диеты отмечает усиление болей в правом по дреберье,тошноту,горечь во рту.Выберите наиболее эффективные желчегонные средства, обладающие одновременно противомикробной активностью:
а) Аллохол, б) Холензим, в) Никодин, г) Отвар пижмы,д) Ксилит
Ответ: а,в

89. с суицидальной целью выпила 20 таблеток феназепама.Через 2 ч после приема препарата доставлена в стационар. Б-я в сознании,но резко заторможена.Проведено промывание желудка.Выберите наиболее оптимальные слабительные средства: а) Глауберова соль, б) Сульфат магния, в) Экстракт коры крушины, г) Бисакодил,
д) Касторовое масло, е) Морская капуста, ж) Вазелиновое масло
Ответ: а,б,д

90. 46 лет поступил в отделение кардиореанимации с острым трансмуральным инфарктом миокарда, возникшим около 5 ч назад.Назначения:анаприлин 20 мг 4 раза в сутки внутрь, гепарин в/в капельно по 10 000 ЕД каждые 4 часа.При этом удалось достигнуть увеличения времени свертывания крови до 18-23 минуты. На 4-й день у б-ного выявлена микрогема турия (22 эритроцита в поле зрения). Какова ваша тактика?
Ответ: Снизить дозу гепарина вплоть до времени свертывания крови не менее 10-12 мин

91. поступил с острым инфарктом миокарда,возникшим 5ч назад.Назначения: анаприлин 20 мг 4 раза в сутки внутрь, гепарин в/в капельно по 10 000 ЕД каждые 4 часа.При этом удалось достигнуть увеличения времени свертывания крови до 18-23 минуты. На следующий день у б-ного диагностирова на правосторонняя нижнедолевая пневмония.Назначена натриевая соль бензилпенициллина (по 1000 000 ЕД каждые 4 ч) в/в.Через 4 ч время свертывания крови составило 8 минут. Какова ваша тактика?
Ответ: Изменить путь введения пенициллина

92. проведена радикальная операция по поводу рака желудка. На 4 сутки после операции при исследовании коагулограммы выявлены гиперкоагуляция и снижение фибринолитической активности крови.Целесообразно ли назначение антикоагулянтов?
Ответ: Антикоагулянты показаны,однако необходим тщательный контроль для предупреждения геморрагического синдрома

93. поступила в стац.с жал-ми на резкую слабость, одышку при ходьбе. При обследовании в анализе крови выявлена анемия (гемоглобин-56 г/л), цветной показатель 1,2,при осмотре языка-глоссит.В пунктате костного мозга выявлен мегалобластный тип кроветворения.Концентрация железа в сыворотке крови в пределах нормы.Диагноз: В 12-дефицитная анамия.Выберите наиболее оптимальный вариант лечения.
Ответ: Вит.В12 в дозе 500 мкг/сут через день, фолиевая кислота в дозе 1,5 мг/сут, сульфат железа(80 мгFe2+) 1 раз в сутки

94. после переохлаждения возникли озноб,повышение Т.тела до 38,6С, кашель с отделением слизисто-гнойной мокроты,боли в правой половине грудной клетки.Клинически и рентгенологически установлен диагноз правосторонней нижнедолевой пневмонии.Б-ной назначено лечение:цефазолин по 0,5 г 2р.в сутки вм,гемодез 400 мл в/в капельно, отхаркивающая микстура по 1ст.л.6р.в сутки.Выберите антиоксидантный препарат,наиболее
эффективно влияющий на процессы свободнорадикального окисления в легких,который следует добавить к проводимой терапии
Ответ: Вит.С

95. по поводу острого нарушения мозгового кровообращения по ишемическому типу давностью 12ч получает реополиглюкин по 400мл в/в капельно
1р.в сутки.Выберите наиболее эффективный в этой ситуации препарат,обладающий антиоксидантными свойствами
Ответ: Церебролизин

96. в течение 5 лет страдает деформирующим остеоартрозом нижних конечностей с выраженными синовитами. В анамнезе отмечает лекарственную
аллергию (на бутадион,гепарин,метиндол,пенициллин,теофиллин).В стаци-онаре б-ной назначены реопирин по 5 мл в/м 1 раз в сутки,гидрокортизона гемисукцинат по 100 мг в полость коленных суставов,тавегил по 0,001 г 2р.в сутки.Через 3 дня у б-ной появились зудящие эритематозные высыпания на коже туловища.Какова наиболее вероятная причина ухудшения состояния?
Ответ: Лекарственная аллергическая реакция

97. подтвержден диагноз ревматоидного артрита.Какие базисные средства для лечения ревматоидного артрита вы можете назначить: а)4,7-хлорхинолоновые препараты (делагил), б) Цитостатики (азатиоприн, циклофосфан и др.), в) Глюкокортикоиды (преднизолон), г) НПВС,
д) Препараты золота (кризанол), е) Салазопиридазин,
ж) Антибиотики (тетрациклины), з) D-пеницилламин,
и) Иммуномодуляторы (левамизол)
Ответ: а,б,д,е,з,и

98. Пациенту с ревматоидным артритом был назначен метотрексат. Как скоро проявится действие метотрексата?
Ответ: Через несколько месяцев

99. с ревматоидным артритом был назначен метотрексат. Какие вы предпримите меры по контролю за безопасностью фармакотерапии метотрексатом у данного пациента: а) Еженедельное проведение общего анализа крови
(лучше 2р.в неделю), б) Проведение анализа крови с определением количества тромбоцитов каждые 3-4 нед,
в) Проведение общего анализа мочи, г)Определение содержания мочевой кислоты, д) Проведение пробы на скрытую кровь в кале, е) Определение содержания трансаминаз,общего билирубина каждые 6-8 нед
Ответ: а,б,в,д,е

100. по поводу ревматизма длительно получает делагил.Какие вы предпримите меры по контролю безопасности терапии делагилом при его длительном применении: а)Общий анализ крови, б)Общий анализ мочи, в)ЭКГ, г)Исследование глазного дна, д)Исследование полей зрения, е)Рентгенологическое исследование органов грудной клетки, ж)Определение количества тромбоцитов, з) Исследование роговицы
Ответ: а,б,в,г,д,ж,з

101. 39 лет ревматоидный артрит, преимущественно суставная форма,2 степени активности.Какие варианты комбинированной терапии целесообразно назначить данному пациенту?
Ответ: Делагил по 0,25 г 3 раза в сутки, преднизолон по 15 мг/сут,кризанол в/м по 1 мл 5% раствора 1 раз в неделю

102. 63 лет страдает сахарным диабетом,принимает глибенкламид.Поступила в отделение с картиной острой правосторонней нижнедолевой пневмонии, подтвержденной рентгенологически.Был назначен хлорамфеникол,на который у б-ной отмечалась аллергическая реакция.Препарат отменили,выбран другой антибиотик-цефтриаксон.Однако при обследовании у б-ной выявлен низкий уровень клиренса креатинина (24 мл/мин),вследствие чего цефтриаксон был отменен.Каким препаратом следует продолжить лечение?
Ответ: Ципрофлоксацин

103. поступила в отделение с ж-ми на боли в правой молочной железе, повышение Т.до 39,5 С.Заболела 3 дня назад,на 10-й день после родов. При поступлении в отделение в верхненаружном квадранте правой молоч-
ной железы обнаружена гиперемия кожи,массивный инфильтрат с флуктуацией в центре.Диагноз:острый правосторонний мастит.Б-ная оперирована. Взят посев отделяемого раны.Определите антибиотик первого выбора
Ответ: Цефазолин

104. поступила в отделение с картиной острого правостороннего мастита.Заболела 3 дня назад,на 10-й день после родов. Б-ная оперирована.
Был назначен цефазолин.После 2-й инъекции препарата через 20 мин появились снижение АД, головокружение, тошнота, рвота, непроизвольное мочеиспускание, судорожный синдром. Какое осложнение развилось у пациентки?
Ответ: Анафилактическая реакция

105. Б-ная 21г.поступила в отделение с картиной острого правостороннего мастита.Заболела 3 дня назад, на 10-й день после родов.Б-ная опери рована.На цефазолин у б-ной отмечалась анафилактическая реакция,препарат был немедленно отменен.При посеве отделяемого раны выделены
стафилококк,образующий пенициллиназу,и гемофильная палочка.Выберите антибактериальный препарат с учетом бактериальной микрофлоры и особенностей фармакокинетики
Ответ: Левомицетин

106. страдает хроническим тонзиллитом и хроническим холециститом.В посеве отделяемого зева и в посеве желчи при обследовании выявлен золотистый стафилококк,образующий пенициллиназу.В анамнезе отмечена аллергия на оксациллин.Б-ной был назначен гентамицин.У б-ного клиренс креатинина составляет 50 мл/мин. Нужно ли корригировать режим дозирования препарата? Если да, то каким образом?
Ответ: Уменьшить кратность введения и снизить дозу

107. 50 лет обратилась с жалобами на общую слабость,жажду,частое мочеиспускание,зуд кожи и наружных половых органов.При осмотре выявлено ожирение (масса тела 96 кг при росте 168 см).Содержание глюкозы в крови 9,9 ммоль/л,в моче 1%, реакция на ацетон отрицательная. Какие гипогликемические препараты оптимальны в данном случае?
Ответ: Бигуаниды

108. 48 лет, поступил с жалобами на давящие боли, появляющиеся во время физической нагрузки, купирующиеся нитроглицерином. 3 года назад перенес инфаркт миокарда. В легких везикулярное дыхание. Тоны сердца приглушены, систолический шум на верхушке, частые экстрасистолы. ЧСС — 92 в мин. АД — 100/60 мм рт. ст. Печень не увеличена, отеков нет. ЭКГ — синусовая тахикардия, рубцовые изменения миокарда, частая желудочковая экстрасистолия. Назначен обзидан 160 мг/сутки, сустак-форте 19,2 мг/сутки, панангин, рибоксин.
Какой побочный эффект вероятен у больного при данной комбинации лекарственных средств?
Ответ: Гипотония, головокружение.

109. Больной М., 52 лет, поступил с жалобами на одышку, сердцебиение, боли в правом подреберье отеки на ногах. В течение 18 лет находится на диспансерном учете с диагнозом «ревматизм». Кожа бледная, акроцианоз, румянец щек. В базальных отделах легких-незвучные мелкопузырчатые хрипы. Границы относительной тупости средца расширены вверх и вправо. Тоны сердца приглушены, аритмичные, на верхушке систолический шум, акцент?? тона на легочной артерии. Пульс-96 в мин. ЧСС-140 в мин. АД — 130/85 мм рт. ст. Живот мягкий, печень на 3-4 см выступает из-под края реберной дуги. Отеки на ногах. Суточный диурез -650 мл. ЭКГ: отсутствует зубец Р, имеются волны «F-F», ритм неправильный. После в/в введения 10 мл 10% р-ра новокаинамида: восстановился синусовый ритм с ЧСС — 72 в мин, больному назначен новокаинамид внутрь по 0,5 г 4 раза в день, дигоксин 0,25 мг 1 табл.
3 раза в сутки, фуросемид 40 мг внутрь в течение 3-х дней. Через 5 дней у больного появилась тошнота, рвота, диарея, головокружение. ЭКГ: ритм синусовый,ЧСС-76 в мин, PQ -0,20 с, QRS-0,1с. Лечащий врач отменил дигоксин и фуросемид и назначил унитиол, препараты калия.
Оцените действия врача.
Ответ: Действия врача правильные, так как не только превышена средняя суточная доза дигоксина, но имеется и взаимодействие с новокаинамидом за связь с белком.

110. У больного Д., 53 лет, диагноз: ИБС, стабильная стенокардия III ФК, постинфарктный кардиосклероз, мерцательная аритмия,ХНК??Б ст. Принимал строфантин, дигоксин, фуросемид, панангин в средних терапевтических дозах. Неожиданно у больного повысилась температура до 38,4°С, появилась кашель, одышка, крепитация в легких справа. На рентгенограмме легких справа в нижней доле определяется участок инфильтрации. К лечению добавлены гентамицин, сульфокамфокаин,супрастин.
Возникновение каких побочных эффектов лечения наиболее вероятно у больного при такой комплексной терапии?
Ответ: При комбинации с фуросемидом наиболее вероятен нефротоксический эффект гентамицина.

111. Больной 28 лет поступил с жалобами на приступы сердцебиения, головную боль, озноб. Во время криза, который развивается 2-4 раза в году, повышается АД до 260/110 мм рт. ст., ЧСС-140 в мин., появляются бледность кожи, жгучие боли в области сердца, пульсация в голове, иногда повышение температуры тела до 38. После приступов полиурия. В межприступный период АД 120/80 мм рт. ст. При обьективном исследовании органической патологии со стороны внутренних органов не обнаружено. В анализах крови и мочи без патологии.
Укажите наиболее эффективный препарат (первый ряд) для купирования криза у больного:
Ответ: Фентоламин.

112. ревматоидным артритом на фоне курсового лечения метотрексатом возникло выраженное носовое кровотечение.Чем оно может быть обусловлено: а)Поражением сосудов носа вследствие основного патологического процесса, б)Повышением ПИ вследствие токсического гепатита,вызванного метотрексатом, в) Увеличением агрегации тромбоцитов под влиянием метотрексата,
г) Медикаментозно обусловленным снижением количества тро мбоцитов, д) Токсическим влиянием метотрексата на сосуды носа
Ответ: г,д

113. У больного К., 62 лет, АГ 1 степени. Последнее ухудшение состояния обусловлено психоэмоциональным стрессом. При осмотре: состояние относительно удовлетворительное, незначительная головная боль. АД-170/100 мм рт.ст. («рабочее» АД-120/70 мм рт.ст), ЧСС-90 в мин. Лечащий врач назначил анаприлин по 60мгсут, верапамил 160 мгсут.
Какие изменения Вы можете ожидать при назначении дополнительно к анаприлину верапамила?
Ответ: Усиление отрицательного дромотропного эффекта.

114. Больной С., 56 лет, по поводу стенокардии принимает нитросорбид (10 мг) 1т х 4р в день. Как изменится тактика проведения антиангинальной терапии, если у больного на фоне лечения нитратами возникнет мозговой инсульт?
Ответ: Отменить нитраты и назначить антиангинальный препарат из другой группы.

115. Больной 42 года с хроническим гломерулонефритом и артериальной гипертензией. При поступлении: АД 200/120 мм рт ст., пульс 75-80 уд в мин, отеки на лице, пояснице,голенях. Общий белок сыворотки крови 3.8 г%, в моче белок 16 г/л.. Укажите препараты, наиболее эффективные для гипотензивной терапии у данного больного:
Ответ: Эналаприл.

116. страдающему упорно рецидивирующим синдромом бронхиальной обструкции врач ввел п/к 1 мл адреналина.Какие проявления токсического
действия адреналина возможны в этой ситуации: а)Возбуждение ЦНС,б)Экстрасистолия, в)Токсическое поражение печени, г)Тахикардия, д)Блокада проведения импульса по проводящей системе сердца.
Ответ: а,б,г

117. 57 лет по поводу посинфарктного ардиосклероза,застойной сердечной недостаточности 2Б степени получает 40 мг фуросемида в/в и 300 мг
верошпирона внутрь.Какую диуретическую терапию Вы назначите б-ному в случае рефрактерности?
Ответ: Фуросемид 80 мг в/в и спиронолактон 300 мг внутрь

118. страдает неатопической бронхиальной астмой,сопровождающейся обильной бронхореей.Пульс 62 в мин.АД 140/80 мм.рт.ст. После назначения атропина сульфата у б-ной вначале отмечалось улучшение состояния-бронхорея резко уменьшилась,однако через 10 дней после начала лечения состояние вновь ухудшилось: появились лихорадка(37,8 С),одышка,кашель с трудно отделяемой мокротой, ЧСС 90 в минуту. Каковы причины подобных изменений состояния больной?
Ответ: Нарушение отхождения мокроты с ее последующим инфицированием

119. Женщина 52 лет страдает гипертонической болезнью?? ст. Принимает резерпин по 1 табл. (0.0001) 3 раза в день. АД нормализавалось через 1 неделю. Через 4 недели регулярного приема появились «голодные» боли в эпигастральной области, в ходе гастроскопии был диагностирован эрозивный дуоденит. Чем Вы обьясните его возникновение?
Ответ: Повышением тонуса n vagus на фоне резерпина и усилением желудочной секреции.

120. Больному 60 лет с ИБС, стабильной стенокардией IV ф.к. назначен кордарон 600 мг/сутки (в качестве антиангинального препарата).
Какие побочные эффекты могут наблюдаться у больного при длительном приеме кордарона?
Ответ: все перечисленное

121. При выборе режима дозирования лекарственных средств на основе Т?
определяют:
Ответ: кратность приема

122. Более точно характеризует скорость выведения лекарственных средств из организма:
Ответ: общий клиренс

123. Связь лекарственных средств с белками плазмы:
Ответ: определяет возможность развития побочных эффектов при сочетании лекарственных средств

124. Величина биодоступности важна для определения:
Ответ: пути введения лекарственных средств*

125. При длительном использовании сильных диуретиков может возникнуть:
Ответ: нарушение толерантности к глюкозе

126. Головокружение, отсутствие ощущения конечностей, затруднения при посадке и вставании без визуального контроля и другие симптомы токсического влияния встречаются у 75% пациентов, которые:
Ответ: получают стрептомицин

127. Передозировка симпатомиметиками вызывает:
Ответ: нарушения ритма

128. Побочные реакции, связанные с антибиотиком моксалактамом, включают в себе следующее:
Ответ: тромбоцитопению

129. Комбинированный прием индометацина и гентамицина наиболее часто вызывает:
Ответ: нарушение функции почек

130. Применение клавулановой кислоты в сочетании с амоксициллином позволяет:
Ответ: расширить спектр действия амоксициллина на штаммы бактерий, производящих бета-лактамазу

131. Больному, длительно получающему дифенин, планируется дополнительно назначить другой антиритмический препарат 1 класса, при назначении какого антиаритмика потребуется увеличение дозировки на 20-30% от стандартной?
Ответ: всех препаратов

132. Одновременный прием внутрь тетрациклина и препаратов Са2+ будет способствовать:
Ответ: снижению всасывания тетрациклина

133. Одновременно назначение хлорамфеникола и аценокумарола может привести:
Ответ: к снижению антибактериальной активности хлорамфеникола

134. При сердечной недостаточности:
Ответ: дофамин вызывает сужение сосудов коры почек в высоких дозах (более 10 мкг/кг/мин)

135. Для артериальной гипертензии характерно:
Ответ: увеличение концентрации натрия в сосудистой стенке

136. Апрессин (гидралазин):
Ответ: вызывает тахикардию

137. Бета-адреноблокаторы вызывают:
Ответ: снижение ЧСС

138. Справедливы такие утверждения об альфа-адреноблокаторах:
Ответ: все верно

139. Бета-1 — адреноблокаторы:
Ответ: избирательно действуют на бета1 -адренорецепторы, препараты безопасны при бронхиальной астме

140. Показаниями к применению бета-адреноблокаторов являются:
Ответ: нарушения сердечного ритма

141. Укажите правильные утверждения:
Ответ: строфантин в значительной степени разрушается в ЖКТ, в связи с чем прием его внутрь нерационален

142. Показания к назначению СГ:
Ответ: ХНК у больных ИБС, постинфарктным кардиосклерозом и постоянной формой мерцательной тахиаритмии

143. Фактор, повышающий риск развития интоксикации СГ:
Ответ: гипокалиемия

144. Состояние, повышающее риск развития интоксикации СГ:
Ответ: гипотиреоз

145. Для уменьшения риска развития толерантности к нитратам следует:
Ответ: делать перерывы между приемами препаратов

146. В случае развития толерантности к сустаку его можно заменить:
Ответ: корватоном

147. Головную боль может вызвать прием:
Ответ: верны ответы А,Б,В

148. Сходным по механизму действия с нитроглицерином является:
Ответ: молсидомин

149. При передозировке каких препаратов может возникнуть ортостатическая гипотония?
Ответ: нитратов

150. Назовите группу антиаритмиков, увеличивающих продолжительность потенциала действия:
Ответ: блокаторы калиевых каналов

151. Какой из перечисленных препаратов оказывает наиболее выраженное отрицательное инотропное действие?
Ответ: дизопирамид

152. Какие из некардиальных побочных эффектов характерны для большинства препаратов 1С класса?
Ответ: нарушение зрения

153. При терапии дизопирамидом может обостриться следующее заболевание:
Ответ: доброкачественная гиперплазия простаты с нарушением мочеиспускания

154. В каких случаях дозировка лидокаина должна быть изменена по сравнению со стандартной?
Ответ: у пациентов с печеночной недостаточностью

155. Укажите методы контроля за эффективностью применения диуретиков при отечном синдроме:
Ответ: все верно

156. Укажите методы контроля за безопасностью применения диуретиков при отечном синдроме:
Ответ: все верно

157. Укажите эффективный и безопасный способ пополнения запасов калия в организме:
Ответ: назначение панангина внутрь по 2 таблетки 3 раза в день

158. Укажите факторы риска возникновения побочных эффектов «петлевых» диуретиков:
Ответ: суточный диурез более 3 л после введения диуретика

159. Укажите начало действия спиронолактона:
Ответ: 4-5 дней

160. Для ощелачивания мочи используют следующие методы, кроме:
Ответ: цитрат калия 3 мг каждые 6 часов

161. Реакция мочи может быть кислой при использовании следующих препаратов, кроме:
Ответ: метионин

162. Отметьте ошибочные положения:
Ответ: нет ошибочных положенией

163. Распределите указанные препараты по степени кумуляции:
Ответ: неодикумарин

164. Отберите утвеждения, которые являются полностью правильными для препарата стрептокиназы:
Ответ: все верно

165. Отберите фактор, вызывающий тромбоз или способствующий тромообразованию:
Ответ: все верно

166. Какой из следующих лекарственных препаратов может уменьшить эффект антикоагулянтов непрямого действия?
Ответ: рифампицин

167. Какие побочные эффекты могут встречаться при применении гепарина?
Ответ: все перечисленное

168. Больному с бронхиальной астмой, длительно получавшему пролонгированные теофиллины, в связи с развитием инфекции мочевыводящих путей назначают ципрофлоксации. В этом случае необходимо:
Ответ: уменьшить дозу теофиллинов на 30%

169. У ребенка, длительно получающего карбамазепин в связи с наличием эпилепсии, развивается бронхообструктивный синдром с дыхательной недостаточностью 2-й ст. При назначении аминофиллина такому пациенту:
Ответ: доза аминофиллина должна быть увеличена в 1.5 раза

170. При назначении теофиллина курильщику:
Ответ: доза должна быть увеличена

171. Укажите препарат, снижающий элиминацию теофиллина при одновременном назначении:
Ответ: циметидин

172. У больному бронхиальной астмой, длительно получавшего теотард, на фоне гриппозной инфекции и лихорадки появилась тошнота, рвота, головные боли, бессонница. Менингиальные симптомы отрицательны. Терапевтическая тактика в данном случае:
Ответ: отменить теотард или снизить его дозу на 50%

173. Побочные эффекты теофиллина могут включать следующие явления, кроме:
Ответ: развития отечного синдрома

174. Специфическим побочным эффектом, возникающим при применении теофиллина у детей 1-го года жизни, является:
Ответ: мелена

175. Укажите ингаляционный глюкокортикостероидный препарат, обладающий наименьшей биодоступностью:
Ответ: флутиказона пропионат

176. Обозначьте ингаляционный глюкокортикостероидный препарат, обладающий наименьшим сродством к глюкокортикостероидным рецепторам легких человека:
Ответ: флутиказона пропионат

177. Укажите препарат, обладающий наибольшей степенью безопасности (по индексу безопасности:)
Ответ: преднизолон

178. Какой из глюкокортикостероидных препаратов в наибольшей мере способствует развитию миопатии?
Ответ: триамцинолон

179. Замедление выделения из организма натрия и воды, усиление выведения калия (минералокортикоидный эффект) в большей степени характерна:
Ответ: гидрокортизону

180. Минералокортикоидная активность отсутствует у:
Ответ: дексаметазона

181. Выберите правильный ответ. Глюкокортикоиды:
Ответ: являются контринсулярными гормонами

182. При проведении пульс-терапии более предпочтителен:
Ответ: метилпреднизолон

183. При назначении на длительный срок предпочтительнее использовать:
Ответ: преднизолон

184. Какой блокатор Н1-гистаминорецепторов противопоказан при анафилактическом шоке?
Ответ: дифенгидрамин (димедрол)

185. Выберите оптимальный блокатор Н1-гистаминорецепторов для лечения аллергического ринита:
Ответ: азеластин (аллергодил)

186. Обозначьте лекарственное средство из группы стабилизаторов мембран тучных клеток в лекарственной форме в виде порошка для ингаляций:
Ответ: кромоглициевая кислота (бикромат)

187. К иммуностимулятору микробного происхождения относится:
Ответ: рибомунил

188. Основное показание для назначения рибомунила заключается в:
Ответ: профилактике рецидивирующих инфекций верхних дыхательных путей

189. Хорошо проникают через гематоэнцефалический барьер следующие антибактериальные препараты:
Ответ: цефалоспорины III генерации

190. Новое поколение макролидных антибиотиков имеет следующие преимущества, кроме:
Ответ: почечный путь экскреции

191. Фторхинолоны отличаются от хинолонов следующими свойствами, кроме:
Ответ: бактериостатическим действием

192. Отметьте, какие утверждения в отношении цефалоспоринов правильны:
Ответ: все верно

193. Последствия приема антибиотиков включают:
Ответ: все верно

194. Укажите препарат выбора при инфекции мочевых путей, вызванной синегнойной палочкой:
Ответ: цефтазидим

195. Какие препараты показаны для лечения хламидийной инфекции мочеполового тракта:
Ответ: ровамицин

196. Укажите препарат с наименее выгодными фармакокинетическими характеристиками:
Ответ: кетоконазол

197. Укажите антимикотический препарат, не подвергающийся метаболизму в печени:
Ответ: флуконазол

198. Укажите антимикотический препарат (из группы аллиламинов), применяемый в первую очередь для лечения дерматомикозов:
Ответ: тербинафин

199. Укажите клиническое состояние, являющееся показанием для проведения монотерапии НПВС:
Ответ: внесуставные ревматические заболевания (миозит, тендовагинит, синовит)

200. Для ацетилсалициловой кислоты характерно:
Ответ: при оральном приеме всасывается в основном из верхнего отдела тонкого кишечника

201. В сравнении с индометацином у ацетилсалициловой кислоты более выражено:
Ответ: антиагрегантное действие на тромбоциты

202. На скорость выведения ацетилсалициловой кислоты и ее метаболитов влияют:
Ответ: уровень рН мочи

203. Желудочно-кишечные осложнения при применении ацетилсалициловой кислоты связаны с:
Ответ: все перечисленное

204. Для фенилбутазона характерно:
Ответ: все верно

205. При взаимодействии индометацина с другими препаратами:
Ответ: уменьшается диуретическая активность фуросемида

206. Какие побочные реакции НПВС коррегирует комплексный препарат артротек (диклофенак натрия + мизопростол)
Ответ: НПВС-гастропатии

207. Какие характеристики парацетамола выдвинули этот препарат на первое место в ряду анальгетиков-антипиретиков?
Ответ: более раннее наступление анальгетического и жаропонижающего эффекта

208. Выберите препарат, селективно ингибирующий циклооксигеназу2:
Ответ: мелоксикам

209. Наилучший анальгезирующий эффект фентанила наблюдается в комбинации с:
Ответ: дроперидолом

210. Назовите противовоспалительное средство пролонгированного действия:
Ответ: пироксикам

211. У Б-ой 52л.развилась картина гипертонического криза 2 типа с ЧСС 62 в мин.АД 200/140 мм.рт.ст.В легких большое кол-во влажных мелкопузырчатых хрипов.С какого препарата необходимо начинать купирование криза:
Ответ: Фуросемид

212. в течение 6 лет страдает сахарным диабетом 1 типа, получает инсулин по 54 ЕД/сут,что поддерживает уровень гликемии в пределах 7,0 ммоль/л.В последнее время в связи с повышением АД до 16090 мм.рт.ст. лечащий врач назначил гипотиазид в суточной дозе 75 мг в комбинации с эналаприлом в дозе 5 мг.Через 10 дней у больного уровень сахара в крови 10,5 ммольл, отмечается ухудшение самочувствия. Что является ведущей причиной изменения уровня сахара в крови?
Ответ: Комбинация эналаприла с гипотиазидом

213. развилась судорожная форма гипертонического криза, состояние тяжелое, цифры АД 200120 мм.рт.ст., ЧСС 120 в мин. С назначения какого препарата необходимо начинать терапию?
Ответ: Диазепама

214. на фоне применения антибиотика цефтриаксона в течение 10 дней развилась картина псевдомембранозного колита. Какой первый шаг алгоритма врачебной помощи?
Ответ: отмена цефтриаксона, назначение ванкомицина или метронидазола

215. по поводу обострения язвенной болезни желудка в комплексе терапии назначен кларитромицин. Назовите основные отличительные признаки препарата от эритромицина.
Ответ: все верно

216. после перенесенной операции на брюшной полости на 4-е сутки развилась левосторонняя нижнедолевая пневмония. Результаты экспресс-анализа показали наличие MRSA, пенициллино- и аминогликозидорезистентных штаммов энтерококков. Препараты выбора:
Ответ: Ванкомицин

217. находится в отделении интенсивной терапии по поводу синегнойной инфекции. Выберите препараты 1 ряда для лечения?
Ответ: Цефтазидим + аминогликозиды

218. 40 лет без сопутствующих заболеваний по поводу внебольничной пневмонии в амбулаторных условиях был назначен спирамицин внутрь по 3 млн. МЕ 2 рс, на 2-е сутки лечения отмечались интенсивные гастралгии, тошнота, однократная рвота. Выберите альтернативный препарат.
Ответ: доксициклин

219. с хроническим обструктивным бронхитом выявлена пневмония средней тяжести, в амбулаторных условиях назначен амоксиклав внутрь по 625 мг 3 рс. На 2-е сутки у больного развилась крапивница, бронхоспазм. Назовите альтернативный препарат для лечения пневмонии.
Ответ: моксифлоксацин внутрь

220. У ВИЧ- инфицированного б.44 лет диагностирована пневмоцистная пневмония. Назовите препарат для лечения?
Ответ: ко-тримоксазол вв 20 мгкгс 4 рс в течение 21 сут

221. У б-ной 28л. отмечаются ежедневные симптомы бронхиальной астмы, частые обострения, частые ночные симптомы, диагностирована тяжелая персистирующая бронхиальная астма. Назовите препараты базисной терапии.
Ответ: ингаляционные глюкокортикоиды (больше 1000 мкг беклометазона дипропионата) + ингаляционные бета-2-агонисты длительного действия

222. К врачу обратилась беременная женщина (срок беременности 6-7 нед.) с клиническими признаками острой пневмонии. Какие группы антибактериальных препаратов разрешены к применению у беременных?
Ответ: цефалоспорины

223. 57 лет по поводу артериальной гипертензии умеренной степени получает в монорежиме ингибитор АПФ — эналаприл. У больного через 2 года приема препарата отмечается недостаточность эффекта. Какой наиболее приемлемый вариант оптимизации терапии?
Ответ: добавление к препарату диуретика (гипотиазида или индапамида)

224. получает антибактериальный препарат по поводу инфекционного процесса. При внутривенной инфузии препарата отмечается реакция в виде выраженного покраснения кожных покровов верхней половины туловища, лица, шеи, симптомы значительно уменьшаются при снижении скорости инфузии. На какой препарат отмечается такая реакция?
Ответ: Ванкомицин

225. У беременной женщины отмечается активация ревматического процесса. Какой препарат из группы антикоагулянтов можно назначить беременной?
Ответ: Гепарин

226. поступил с ж-ми на повышение АД до цифр 15090 мм.рт.ст. на фоне психоэмоционального перенапряжения, сердцебиение, тревогу, нарушение сна. Год назад выявлен сахарный диабет 2 типа, получает манинил. Назовите препарат выбора для лечения АГ.
Ответ: Атенолол

227. в связи с симптомами стенокардии и нарушения ритма назначены препараты: анаприлин 200 мгс и верапамил 240 мгс в течение длительного времени. Какие возможны побочные реакции?
Ответ: Развитие а-в блокад, брадикардия

228. Женщина 34 лет принимает эстрогенсодержащие контрацептивные препараты. Лечащий врач назначил доксициклин в дозе 200 мгс в течение 2 недель. Какое вероятное взаимодействие ожидается?
Ответ: Снижается эффект контрацепции

229. Анестезиолог при введении кетамина с профилактической целью назначил больному диазепам. Профилактика какого состояния проводится таким образом?
Ответ: Постнаркозных галлюцинаций

230. 46 лет поступил с картиной острого деструктивного аппендицита. Препарат выбора для антибиотикопрофилактики?
Ответ: Цефазолин

231. К врачу обратился больной с жалобами на кашель, повышение температуры до 39 С, боли в грудной клетке. Диагностирована правостронняя бронхопневмония. Назначен препарат в течение 3 суток, обладающий постантибиотическим эффектом. Назовите препарат выбора.
Ответ: Азитромицин

232. по поводу острого правостороннего пиелонефрита назначен цефазолин по 2 гс в течение 10 дней. Назовите наиболее типичную ошибку при выборе данного антибиотика
Ответ: Недостаточно высокая активность в отношении грамотрицательной флоры

233. одновременно получает в течение 14 дней фторхинолоновый антибиотик — офлоксацин по поводу инфекции мочевыводящих путей и диклофенак натрия в связи с суставным синдромом. Какое вероятное взаимодействие ожидается?
Ответ: повышение риска возбуждения ЦНС и развития судорог

234. после переохлаждения возникли озноб,повышение Т.тела до 38,6С, кашель с отделением слизисто-гнойной мокроты,боли в левой половине грудной клетки.Клинически и рентгенологически установлен диагноз левосторонней нижнедолевой пневмонии.Б-ной назначено лечение:цефазолин по 1 г 2р.в сутки вм,гемодез 400 мл в/в капельно, отхаркивающая микстура по 1ст.л.6р.в сутки. На 3-е сутки отмечалась реакция в виде крапивницы, кожного зуда. Выберите антибактериальный препарат для замены?
Ответ: Спирамицин

235. При проведении наркоза анестезиолог с целью антибиотикопрофилактики назначил антибактериальный препарат. У больного развилась остановка дыхания. Какой препарат использовался?
Ответ: Гентамицин

236. в течение 10 лет страдает деформирующим остеоартрозом нижних конечностей с выраженными синовитами. В анамнезе отмечает лекарственную аллергию на бутадион. В отделении б-ной назначены реопирин по 5 мл в/м 1 раз в сутки.Через день у б-ной появились зудящие эритематозные высыпания на коже туловища.Какова наиболее вероятная причина ухудшения состояния?
Ответ: Лекарственная аллергическая реакция

237. по поводу нарушения ритма сердца назначен новокаинамид, одновременно в связи с сезонным аллергическим ринитом назначен цетиризин. Какое вероятное взаимодействие ожидается?
Ответ: тяжелые формы аритмии (типа пируэта)

238. Пациенту с СКВ был назначен метотрексат. Через какое время ожидается стабильное проявление лечебного эффекта?
Ответ: бірнеше айдан кейін

239. с гипертоническим кризом был назначен препарат фозиноприл. Несмотря на прием адекватной дозы препарата не отмечено снижение уровня АД в ближайшие минуты и часы. Назовите причину.
Ответ: При кризах не используются депо-препараты

240. с целью плановой терапии АГ был назначен апрессин в течение длительного времени. У больного через месяц применения препарата стали отмечаться сердцебиение, стенокардитические боли, снижение эффекта от лечения. Назовите основную причину развившихся явлений
Ответ: Апрессин не назначается для плановой терапии АГ

241. 42 лет поступил в отделение интенсивной терапии по поводу тяжелого гипертонического криза. В течение 5 суток внутривенно инфузионно вводился нитропруссид натрия. На 6-е сутки у больного развилась картина интоксикации в виде неукротимой рвоты, снижения функции сердечно-сосудистой, дыхательной и выделительной систем. Назовите основную причину развившегося состояния.
Ответ: Передозировка препарата (накопление тиоцианатов в крови)

242. 54 лет страдает сахарным диабетом 1 типа,принимает пролонгированный инсулин.Поступила в отделение с картиной острой правосторонней нижнедолевой пневмонии, подтвержденной рентгенологически.Был назначен цефтриаксон,на который у б-ной отмечалась аллергическая реакция.Препарат отменили,выбран другой антибиотик- ципрофлоксацин в сочетании с амикацином.Однако при обследовании у б-ной выявлен низкий уровень клиренса креатинина (30 мл/мин),вследствие чего амикацин был отменен.Каким препаратом следует продолжить лечение?
Ответ: Спирамицин

243. поступила в отделение с ж-ми на боли в низу живота, повышение Т.до 39,5 С.Заболела 2 дня назад,на 6-й день после родов. При гинекологическом осмотре — картина острого послеродового эндометрита. Результаты бакпосева: золотистый стафилококк, образующий пенициллиназу, протей. Определите антибиотик первого выбора
Ответ: Цефепим

244. 25 л.поступила в отделение с картиной острого правостороннего пиелонефрита.Заболела 3 дня назад после переохлаждения. Был назначен цефазолин.После 2-й инъекции препарата через 10 мин появились снижение АД, головокружение, тошнота, рвота, непроизвольное мочеиспускание, судорожный синдром. Какое осложнение развилось у пациентки?
Ответ: Анафилактическая реакция

245. Б-ная 28 л.поступила в отделение с картиной острого правостороннего мастита.Заболела на 12-й день после родов.Б-ная оперирована.На цефазолин у б-ной отмечалась анафилактическая реакция,препарат был немедленно отменен.При посеве отделяемого раны выделены стафилококк,образующий пенициллиназу,и кандиды.Выберите антибактериальный препарат с учетом бактериальной микрофлоры и особенностей фармакокинетики
Ответ: Оксациллин + флуконазол

246. страдает хроническим холециститом.В посеве желчи при обследовании выявлены золотистый стафилококк и кишечная палочка. В анамнезе отмечена аллергия на оксациллин. Препараты выбора.
Ответ: Цефтриаксон

247. 58 лет обратилась с жалобами на общую слабость,жажду,частое мочеиспускание,зуд кожи и наружных половых органов.При осмотре: масса тела 56 кг при росте 168 см).Содержание глюкозы в крови 12,3 ммоль/л,в моче 1,5%, реакция на ацетон отрицательная. Какие гипогликемические препараты оптимальны в данном случае?
Ответ: Препараты сульфонилмочевины

248. 53 лет, поступил с жалобами на сердцебиение, перебои, временами одышку. Эти явления стали беспокоить после перенесенного инфаркта миокарда 2 года назад. Прием новокаинамида в течение 3 месяцев принес значительное облегчение. Однако в последнее время самочувствие ухудшилось. Дальнейшая тактика ведения больного.
Ответ: полное обследование и выбор препарата

249. 33 лет по поводу послеоперационного гнойного перитонита получает комбинацию препаратов: цефтриаксон + амикацин + метронидазол. В анамнезе — желчнокаменная болезнь. Назовите препарат, который не должен назначаться больному.
Ответ: Цефтриаксон

250. 45 лет по поводу кандидозной пневмонии был назначен флуконазол вв в течение 3 дней, затем внутрь. На 4-е сутки лечения отмечена реакция в виде сильных головных болей, тошноты. Решено было заменить препарат на кетоконазол. Оцените адекватность тактики.
Ответ: Кетоконазол не является адекватной заменой в силу невыгодных фармакокинетических характеристик

251. 42 лет, поступил с жалобами на выраженное сердцебиение, повышение цифр АД до 240140 мм.рт.ст. При обследовании выявлено значительное повышение в крови уровня катехоламинов. Назовите препарат выбора для купирования криза.
Ответ: фентоламин

252. 50 лет, поступил с картиной острого левостороннего пиелонефрита. Были назначены цефазолин + гентамицин в среднетерапевтических дозах. У больного при обследовании клиренс креатинина — 50 млмин. Какие вероятные последствия терапии?
Ответ: Риск нефротоксичности

253. 48 лет с пароксизмальной над- и желудочковой тахикардией был назначен кордарон вв в первые сутки, затем внутрь. При обследовании выявлено нарушение функции щитовидной железы, а-в блокада 2-3 степени. Дальнейшая тактика.
Ответ: Отмена препарата, назначение новокаинамида

254. 50 лет находится в отделении кардиореанимации по поводу острого инфаркта миокарда, получает комплекс терапии. Какие основные параметры мониторинга при назначении прямых антикоагулянтов?
Ответ: АЧТВ, время свертывания крови, моча на эритроциты

255. В отделении находится больной после операции по поводу аппендицита. Лечащий врач назначил гентамицин по 80 мг 3 рс вм. Пациент 2 года назад перенес острый гломерулонефрит, в настоящее время клиренс креатинина составляет 50 млмин. Какая коррекция требуется?
Ответ: замена на препарат, не обладающий нефротоксичностью

256. 56 лет, получает дигоксин по 0,25 гс в течение последнего года. В настоящее время стало отмечаться повышение цифр АД до 180110 мм.рт.ст. Лечащим врачом назначен лизиноприл в дозе 10 мгс. Через 3 месяца у пациента диагностирована дигиталисная интоксикация. Дальнейшая тактика.
Ответ: назначение гипотензивного препарата другого ряда

257. 53 лет диагностирован кандидозный и аспергиллезный менингит. Препараты выбора.
Ответ: Амфотерицин В

258. 58 лет, получающего длительное время глюкокортикоиды в комплекс терапии был включен синтетический антибиотик широкого спектра в связи с инфекцией репродуктивной сферы. На 14 сутки совместного применения отмечено тяжелое осложнение в виде разрыва ахиллова сухожилия. Назовите антибиотик, который в сочетании с ГКС вызвал данное осложнение.
Ответ: Левофлоксацин

259. в отделение поступил Б.43 лет с картиной острой хламидийной пневмонии. Назовите препараты выбора.
Ответ: Ровамицин

260. У больного 24 лет диагностирована неосложненная форма гонореи. Назовите препарат выбора.
Ответ: Цефтриаксон

261. Выберите правильное утверждение: а)биодоступность-количество ЛС,поступающее в системный кровоток,выраженное в процентах от введенной дозы,б)биодоступность определяется величиной адсорбции ЛС в ЖКТ и выра-
женностью эффекта первого прохождения через печень.в)биодоступность определяют по формуле: F = AUC (в/м или внутрь)/AUC (в/в).г)биодоступность ЛС при внутримышечном введении определяется степенью его всасывания и биотрансформации в организме.
Ответ: а, б, в

262. страдает неатопической бронхиальной астмой,сопровождающейся обильной бронхореей.Пульс 62 в мин.АД 140/80 мм.рт.ст.Какие препараты более предпочтительны?
Ответ: Атровент

263. Механизмы всасывания ЛС в кишечнике:
а) пассивная диффузия, б) фильтрация, в) активный транспорт,г) облегченный транспорт, д) пиноцитоз:
Ответ: а, д

264. поступил с ж-ми на изжогу,боли в эпигастральной области натощак, купируемые приемом натрия гидрокарбоната.При ФЭГДС выявлена язва (0,5см в диаметре) в ампуле 12 п.к.РН-метрия желудочного сока: кислотообра-
зующая функция средней интенсивности с низкими щелочными резервами,холинергический тип рецепции. Диагноз: язвенная болезнь 12 п.к.в стадии обострения. Выберите наиболее эффективное и безопасное ЛС и определите режим его дозирования:
Ответ: Пирензепин до еды по 0,05 г 3 раза в сутки в течение 2дней, затем по 0,05 г 2 раза в сутки

265. выявлена дискенезия желчного пузыря по гипертоническому типу. Выберите оптимальный вариант лечения.
Ответ: Но-шпа по 1-2 табл.3 раза в сутки, отвар бессмертника по 1/2 стакана за 30 мин до еды

266. страдает хрон.холецистопанкреатитом в течение 5 лет.За последнюю неделю после нарушения диеты отмечает усиление болей в правом по дреберье,тошноту,горечь во рту.Выберите наиболее эффективные желчегонные средства, обладающие одновременно противомикробной активностью:
а) Аллохол, б) Холензим, в) Никодин, г) Отвар пижмы,
д) Ксилит
Ответ: а,в

267. с суицидальной целью выпила 20 таблеток феназепама.Через 2 ч после приема препарата доставлена в стационар. Б-я в сознании,но резко заторможена.Проведено промывание желудка.Выберите наиболее оптимальные слабительные средства: а) Глауберова соль, б) Сульфат магния, в) Экстракт коры крушины, г) Бисакодил,
д) Касторовое масло, е) Морская капуста, ж) Вазелиновое масло
Ответ: а,б,д

268. 46 лет поступил в отделение кардиореанимации с острым трансмуральным инфарктом миокарда, возникшим около 5 ч назад.Назначения:анаприлин 20 мг 4 раза в сутки внутрь, гепарин в/в капельно по 10 000 ЕД каждые 4 часа.При этом удалось достигнуть увеличения времени свертывания крови до 18-23 минуты. На 4-й день у б-ного выявлена микрогематурия (22 эритроцита в поле зрения). Какова ваша тактика?
Ответ: Снизить дозу гепарина вплоть до времени свертывания крови не менее 10-12 мин

КАТЕГОРИИ

ПОПУЛЯРНЫЕ СТАТЬИ

© 2024 «kingad.ru» — УЗИ исследование органов человека